You are on page 1of 93

LET’S CRACK IT!

Environment
Current Affairs
for UPSC Prelims 2024
January 2022 to December 2023

Volume -I

PT-730
Table of Contents

Biodiversity Conservation 7
1. Eco-Sensitive Zones (ESZ) 7
2. Biodiversity Heritage Sites (BHSS) - Ladakh first BHSS 8
3. Biological diversity (Amendment) Bill, 2021 9
4. Cheetah Reintroduction 10
5. Bird life international 11
6. Invasive Species 11
7. Loktak lake 11
8. Mangroves for Future (MFF) 12
9. Biodiversity Heritage Sites (BHSS) - Ladakh first BHSS 12
10. Tiger conservation foundation 13
11. Biological Diversity (Amendment) Act, 2021 13
12. Biodiversity and Ecosystems services Index 14
13. Central Asian Flyway 14
14. Status of Leopard in India 2018 15
Climate Change 16
15. Glacial lake outburst flood (GLOF) 16
16. Flash flood guidance system (FFGS) 19
17. Coral Reef Protection 21
18. Bharat Stage Norms 21
19. Climate Smart Cities Assessment Framework 2.0 22
20. Carbon Pricing 22
21. Greater Maldives Ridge 23
22. Geomagnetic Storm 24
23. Karewas 25
24. Karakoram Anomaly 26
25. Blue Flag Certification 26
Environmental Pollution 27
26. Ammonia pollution 27
27. Zero liquid discharge (ZLD) 27
28. Microplastics 28
29. E-Waste Management 28
Summit, Conferences, and Conventions 29
30. Bunker Convention 29
31. South Asian Climate Outlook Forum (SASCOF) 30
32. Global Platform for Disaster Risk Reduction 2022 (GP2022) 30
33. International Solar Alliance (ISA) 31
34. St. Petersburg declaration 32
35. The UN Water Conference 33
36. Carbon Pricing Leadership Coalition 34
37. High Ambition Coalition for Nature and People 35
38. UNCCD: Land Degradation Neutrality Fund 35
39. 5 International Marine Protected Areas Congress
th
37
40. UN Ocean Conference 38
41. Stockholm Conference, 1972 38
42. Convention on Biological Diversity (CBD) 40
43. Clean Air Fund 41
44. OECM Status 42
45. Clydebank Declaration 43
National and Global Initiatives 44
46. National river conservation plan (NRCP) 44
47. National Water Mission 44
48. Coalition for Infrastructure (CDRI) 45
49. E-flow norms for river Ganga 46
50. Innovation Roadmap of the Mission Integrated Biorefineries (IRMIB) 47
51. Mission LiFE (Lifestyle for Environment) 47
52. Carbon Border Adjustment Mechanism (CBAM) 48
53. Energy Conservation (Amendment) Act, 2022 49
54. National Mission on Natural Farming 50
55. E-amrit portal 51
56. National mission for enhanced energy efficiency (NMEEE) 51
57. Species recovery programme 52
58. Antarctic treaty 52
59. Seea- ecosystem accounting 53
60. Conservation assured tiger standards(CA|TS) 53
61. Aravali Green wall project 53
62. Angria Bank 54
63. International Big Cat Legislation 54
64. Graded Response Action Plan 54
65. India’s First Sovereign Green Bond Framework 55
66. Sustainable Cities Integrated Approach Pilot (SCIAP) Project 57
67. Ocean Thermal Energy 58
68. Dam safety Bill , 2019 60
69. Rooftop Solar Programme 61
70. National Hydrogen Mission- Hydrogen Energy 62
71. India’s First Geothermal Energy Project. 64
72. Seaweeds 65
73. Long-Term Low Emission Development Strategy 66
74. Global Standards on nature based solutions 68
75. BIOFIN 68
76. Green Voyage 2050 Project 69
Ranks & Reports 70
77. Global Methane Assessment 70
78. Climate and Clean Air Coalition 70
79. State Energy & Climate Index 71
80. State of India’s birds 2020 report 71
81. Emissions gap report 2021 72
82. Living planet report 2020 72
83. Intergovernmental Panel on Climate Change: Sixth Assessment Report 72
84. IPCC: Asian Waterbird Census 73
85. Cloud Forest Assets 74
86. State of climate services report 2021 75
87. Global assessment on marine litter and plastic pollution 76
Miscellaneous 79
88. Mawmluh Cave 79
89. International Year of Millets (IM) 2023 79
90. Direct-seeded Rice (DSR) 79
91. Energy efficiency services limited (EESL) 80
92. Ethanol blending 80
93. Limits to growth 80
94. Edgar food 81
95. Environmental dna metabarcoding (edna) 81
96. National tiger conservation authority (NTCA) 81
97. Invasive Species 82
98. The Indian Antarctic Act 2022 82
99. Anatolian Plateau Issue 83
100. Illegal Wildlife Trafficking 84
101. Sea Sparkle 85
1 Environment

Biodiversity Conservation

y The Supreme Court has directed that every protected forest, national
park and wildlife sanctuary in the country should mandatorily have
a minimum 1 km ESZ from their demarcated boundaries.
⚪ The directions were passed in reference to applications
filed under the TN Godavarman Thirumalpad versus Union
of India case.

The Judgement
y Applicability: All such states/UTs where the minimum ESZ is not
prescribed.
y Extended boundary shall prevail if the existing ESZ goes beyond 1
km buffer zone or if any statutory instrument prescribes a higher
limit.
y No new permanent structure will be allowed within the ESZ.
y Mining within national wildlife sanctuaries or national parks is not
permitted.
y Activity already being undertaken, within the 1km or extended
ESZ, but lying outside the ambit of prohibited activities may
continue with permission.
1. Eco-Sensitive y The Principal Chief Conservator of Forests of each State/UT
Zones (ESZ) directed to make a list of subsisting structures at the ESZs and
submit a report to the court within 3 months.
y Requirements could be diluted if there is “overwhelming public
interest”.

Eco-Sensitive Zones (ESZs)


y Ecologically important and fragile areas around protected areas
designated under the Environment Protection Act (EPA), 1986
y They are designated as ESZ to protect them from industrial
pollution and unregulated development.
y Notified by: The Central Government in exercise of the Powers
conferred by section 3 of the EPA, 1986.
y Aim of ESZs
⚪ Create some kind of “Shock Absorber” for the Protected Areas
(PAs).
⚪ Act as a transition zone from areas requiring higher protection
to those requiring lesser protection.
⚪ Regulate human activities, rather than prohibit, around PAs.
y Extent of ESZs as per 2011 guidelines: Generally, width of up to 10
kms around a Protected Area.

Environment 7
⚪ Also includes Sensitive corridors, connectivity and ecologically
important patches, crucial for landscape linkages, even
beyond 10 kms width.
y Distribution of an area of ESZ and the extent of regulation may not
be uniform all around and are kept specific to the Protected Area.

Biodiversity Heritage Sites (BHSS)


y They are well defined areas that are unique, ecologically fragile
ecosystems which comprise any of: “species richness, high
endemism, presence of rare, endemic and threatened species,
keystone species, species of evolutionary significance, wild
ancestors of domestic/cultivated species or landraces or their
varieties, past pre-eminence of biological components represented
by fossil beds and having cultural or aesthetic values.”
y Notified by the state government in consultation with local bodies
under Section 37 of Biological Diversity Act 2002.
y Present Status: 35 BHS in India including Majuli, Unakoti caves,
Amarkantak, etc.
⚪ Recently, Yaya Tso Lake of Ladakh was included in the list, the
first BHS of Ladakh.

New Biodiversity Heritage sites:


y Yaya Tso Lake
⚪ The lake is located at an altitude of 4820m above sea level.
⚪ Also known as “Birds Paradise”, it is a very important nesting
2. Biodiversity
and breeding site for a variety of birds such as the bar-headed
Heritage Sites
goose, black-necked crane and the brahminy duck.
(BHSS) - Ladakh
y Gandhamardan Hill (Gandhamardan Reserve Forest)
first BHSS
⚪ Gandhamardan Hill Range in Bargarh and Balangir district of
Odisha has been declared as Biodiversity Heritage sites.
⚪ It is known for the following:
 Medicinal plants and an Ayurvedic paradise of Odisha.
 Two historical monuments: Nrusinghanath Temple located
on the northern slope and Harishankar Temple situated on
the southern slope.
 The hill shrine has been mentioned by Hiuen Tsang as a
Buddhist heritage site named Parimalagiri.
y West Bengal has notified four new BHS
⚪ Char Balidanga and state Horticulture Research and
Development Station campus
 Char Balidanga comprises two islands on Hooghly with
tropical riverine vegetation of tall grasses and swampy flat
lands.
⚪ Namthing Pokhar: It is a Himalayan natural wetland.
⚪ Amkhoi Fossil Park: It has unique geological and paleo-
botanical features and is the first wood fossil park in West
Bengal.

8 Environment
Biological Diversity Act 2002
y It was enacted to fulfill India’s obligations under the Convention
of Biological Diversity of 1992 and Nagoya Protocol on Access
and Benefit Sharing.
⚪ It was enacted to provide for the conservation of biological
diversity, sustainable use of its components and fair and
equitable sharing of the benefits arising out of the use of
biological resources and knowledge.
y It provides for a decentralized three-tiered mechanism for
implementation of the Act.
⚪ The National Biodiversity Authority (NBA) at the national level
⚪ State Biodiversity Boards (SBBs) at the state level
⚪ Biodiversity Management Committees (BMCs) at the local level.
 Biodiversity Management Committees (BMC) prepare
People’s Biodiversity Registers which keep a record of all
flora and fauna including details of traditional knowledge
available in their region.

Key Provisions of the Proposed Amendments


y Extension of Exemptions: Registered AYUSH medical Practitioners,
people accessing codified traditional knowledge, cultivated
3. Biological
medicinal plants and its products, people who are practicing
diversity
indigenous medicine including Indian systems of medicine are
(Amendment)
exempted from giving prior intimation to SBB for accessing
Bill, 2021
biological resources for certain purposes.
y Simplifying access to biological resources and intellectual property
rights (IPR):
⚪ Certain entities must seek approval from the NBA for
obtaining biological resources that have been changed to any
foreign-controlled company registered in India.
⚪ Applicants can now obtain NBA’s approval before the grant of
IPR and not before applying for IPR.
⚪ It seeks to exempt registered AYUSH medical practitioners and
people accessing codified traditional knowledge, among others,
from giving prior intimation to State biodiversity boards for
accessing biological resources for certain purposes.
y Reduction of certain offences: The provision which made
offences under the act cognizable and non- bailable was
deleted. However, penalties may extend up to ₹1 crore.

y States can declare Threatened species: The Central government


can delegate the power to notify any species which is near-
extinction as a threatened species to the state government.
⚪ o However, before notifying any threatened species, the state
government must consult the NBA.

Environment 9
y Expansion of NBA: 11 additional members to be added to NBA,
including:
⚪ 6 ex-officio members dealing with wildlife, forestry research,
and Panchayati Raj
⚪ 4 representatives from SBBs (on a rotational basis), and
⚪ 1 Member-Secretary (must have experience in biodiversity
conservation), who will be the chief coordinating officer of
the NBA.
y Changes in Definitions: Several terms such as ‘bio-utilisation’
have been removed, and others such as ‘bio-survey’ have been
redefined.

Action Plan for Reintroduction of Cheetah


y It was launched at the 19th meeting of the National Tiger
Conservation Authority (NTCA).
y The Action Plan aims to establish a viable cheetah metapopulation
in India that allows the cheetah to perform its functional role
as a top predator and provides space for the expansion of
the cheetah within its historical range thereby contributing to its
global conservation efforts.
y As per the action plan, a cohort of around 10-12 young African
Cheetahs- a different subspecies (Acinonyx Jubatus) will be sent
from Namibia or South Africa to the grassland habitats that
the Asiatic cheetahs occupied in the past as a founder stock
during the first year.

Why is India reintroducing African Cheetah instead of Asiatic Cheetah?


y The locally extinct cheetah-subspecies of India is found in
Iran and is categorized as critically endangered.
4. Cheetah
⚪ However, it is not possible to source the critically endangered
Reintroduction
Asiatic cheetah from Iran without affecting this sub-species.
y Hence, India will source cheetahs from Southern Africa, which can
provide India with substantial numbers of suitable cheetahs for
several years.

Where will cheetahs be relocated in India?


y Kuno National Park (NP) in Madhya Pradesh will get 13 cheetahs,
who can co-exist with leopards.
⚪ Move is being described as the world’s largest intercontinental
animal translocation.
⚪ KNP is part of Sheopur-Shivpuri deciduous open forest
landscape and is estimated to have a capacity to sustain 21
cheetahs.
⚪ It has a suitable habitat and adequate prey base.
⚪ Kuno also offers the prospect of housing four big cats of India -
tiger, lion, leopard and cheetah - and allowing them to coexist
as in the past.

10 Environment
Importance of relocation
y Cheetah’s conservation will revive grasslands and its biomes and
habitat.
⚪ Grasslands are a hugely neglected habitat despite the largest
number of Schedule I protected animals under Wildlife
Protection Act residing in these grasslands.

About Cheetah:
y They are the fastest mammals on land.
y Cheetahs don’t need much water and can survive in dry forests,
grasslands, open plains and desert regions.
y Cheetah is the only large carnivore that got completely wiped out
from India in 1952, mainly due to over-hunting and habitat loss.

Important Bird Areas (IBAs)


y Declared under: IBA programme of Birdlife International
y Aim: To identify, monitor and protect a global network of IBAs
for conservation of the world’s birds and associated biodiversity
y Serve as conservation areas for protection of birds at the global,
regional or sub-regional level
y Release State of the Worlds Birds Reports
5. Bird life
international
Criteria according to Birdlife International:
y Hold significant numbers of one or more globally threatened bird
species
y Hold a suite of restricted-range species or biome-restricted
species
y Have exceptionally large numbers of migratory or congregatory
birds.

Invasive Species
y An organism that causes ecological or economic harm in a new
environment where it is not native.
6. Invasive Species y For E.g.: In India, Parthenium (came from wheat imported from
the U.S. in 1950s) and lantana (brought by British as ornamental
plants from South America) threatens more than 40% of
India’s tiger reserves.

Loktak Lake was recently added


to the approved geo-tourism or
geo-heritage sites.
7. Loktak lake y Loktak Lake is the largest freshwater lake, located in Imphal,
Manipur.
y It is the only floating national park in the world
y The Loktak lane is for endemic flora and fauna:

Environment 11
⚪ It is known for its floating islands called ‘Phumdis’ and Keibul
Lamjao National Park.
⚪ It is the last natural habitat of Sangai Deer, an endangered
deer, also known as ‘dancing deer’.
⚪ Other Species of Loktak Lake: Hog Deer, Otter, a host of
waterfowls and migratory birds.

Mangroves for Future


y It is a partner-led initiative to promote investment in coastal
ecosystem conservation for sustainable development.
⚪ It provides a platform for collaboration among the many
different agencies, sectors and countries which are addressing
challenges to coastal ecosystem and livelihood issues with
the goal to promote an integrated ocean-wide approach to
coastal management and to build the resilience of ecosystem-
dependent coastal communities.
y This work is focused on Asia in Bangladesh, Cambodia, India,
Indonesia, Maldives, Myanmar, Pakistan, Seychelles, Sri Lanka,
Thailand and Vietnam.
y MFF is inclusive of all types of coastal ecosystem, such as coral
reefs, estuaries, lagoons, sandy beaches, seagrasses and wetlands.
8. Mangroves for
Future (MFF) Other initiatives
y MISHTI (Mangrove Initiative for Shoreline Habitats & Tangible
Incomes) scheme aimed at mangrove plantation along the coastline
and on salt pan lands.
⚪ Mangrove plantations will be taken up through the
convergence of the job guarantee scheme MGNREGS,
Compensatory Afforestation Fund, and other funding sources.
y Sustainable Aquaculture in Mangrove Ecosystem (SAIME) Initiative
for sustainable shrimp cultivation provides hope for mangrove
restoration in Sundarbans.
⚪ SAIME is a community-based pilot project in West Bengal under
which farmers are planting mangrove trees around shrimp
ponds.
⚪ Generally, mangrove forests are cleared to cultivate shrimps in
the areas.

y Well defined areas that are unique, ecologically fragile ecosystems,


spread over terrestrial, coastal and inland and marine waters
9. Biodiversity having rich biodiversity.
Heritage Sites y They are notified by State government in consultation with local
(BHSS) - Ladakh bodies
first BHSS ⚪ State Biodiversity Boards (SBB) may invite suggestion for
declaration of BHSs through community institution such as
Gram Sabhas, Panchayats, urban wards, tribal councils, et

12 Environment
⚪ Presently, India has 35 BHS including Majuli, Unakoti, Silachari
Caves, Amarkantak etc

Tiger conservation foundation


y Tiger Conservation Foundation is another innovative mechanism
mandated under the Wildlife (Protection) Act, 1972 for facilitating
tiger conservation by involving local communities through eco-
development process.
y The area of operation of the TCF shall be the tiger reserve and
its adjoining landscape, forming the impact zone with possible
10. Tiger corridor value for dispersal of wild animals from the tiger reserve.
conservation
foundation Eco-bridges for tigers
y Eco-bridge has been conceptualized by the National Board for
Wildlife and Wildlife Institute of India headed by the Prime Minister.
y Telangana became the first state in India to have eco-friendly
bridges for the movement of tigers.
⚪ The eco-bridge has been made over a canal cutting across a
tiger corridor linking the Tadoba-Andhari Tiger Reserve (TATR)
of Maharashtra with the forests in Telangana.

Biological Diversity Act, 2002


Aim:
y To fulfill India’s obligations under the Convention of Biological
Diversity and Nagoya Protocol on Access and Benefit Sharing.
⚪ It provides for a decentralized three-tiered mechanism for
implementation of the Act.
y Biodiversity Management Committees (BMC) are responsible
for preparing People’s Biodiversity Registers which keep a record
of all flora and fauna including details of traditional knowledge
11. Biological available in their region.
Diversity
(Amendment) Biological Diversity (Amendment) Act, 2021
Act, 2021 Objectives for the amendment
y Reduce pressure on wild medicinal plants by encouraging their
cultivation
y Encourage Indian system of medicine
y Facilitate more foreign investments in the area of biological
resources in the context of research, patent and commercial
utilization,
y Facilitate fast-tracking of research, patent application process
transfer of research results while utilising the biological resources
available in India.

Environment 13
The Intergovernmental Science-Policy Platform on Biodiversity and
Ecosystem Services (IPBES) has gathered at Bonn, Germany, for the
body’s 10th plenary.

Intergovernmental Science-Policy Platform on Biodiversity and


Ecosystem Services (IPBES)
y It is an independent intergovernmental body established in 2012.
y It provides policymakers with objective scientific assessments
about the state of knowledge regarding the planet’s biodiversity,
ecosystems and the benefits they provide to people, as well as
the tools and methods to protect and sustainably use these vital
natural assets.
y This independent body was inspired by the Intergovernmental
Panel on Climate Change (IPCC) and the Millennium Ecosystem
Assessment.
y It is not a United Nations body. However, at the request of the
IPBES Plenary and with the authorisation of the UNEP Governing
Council in 2013, the United Nations Environment Programme
12. Biodiversity and
(UNEP) provides secretariat services to IPBES.
Ecosystems
y India is a member country of this organisation.
services Index
IPBES Report:
The Report identifies five broad categories of ‘practices’ in the use of
wild species: Fishing; Gathering; Logging; Terrestrial Animal Harvesting
(including hunting); and Non-extractive practices, such as observing.

Key Findings:
y Dependence on Biodiversity
⚪ Around 70% of the world’s poor are directly dependent on wild
species. 50,000 wild species of plants, animals, fungi and algae
meet the needs of billions of people globally.One in five people
rely on wild species for income and food.
y Threat to Biodiversity
⚪ The global rate of species extinction is already at least tens to
hundreds of times higher than the average rate over the past 10
million years and is accelerating.
y Unsustainable gathering is one of the main threats for several plant
groups, notably cacti, cycads, and orchids as well as other plants.

Eleven countries have collaborated to strengthen conservation efforts


for migratory birds and their habitats in the Central Asian Flyway (CAF).
y The meeting was conducted by The Ministry of Environment, Forest and
Climate Change in collaboration with the United Nations Environment
13. Central Asian
Programme/Convention on Migratory Species (UNEP/CMS).
Flyway
About:
y Central Asian Flyway (CAF) covers a large area of Eurasia between
the Arctic and Indian Oceans.

14 Environment
y This flyway comprises several important migration routes of birds.
y Including India, there are 30 countries under the Central Asian
Flyway.
y At the 13th meeting of the Conference of Parties (CoP) to the
Convention on Migratory Species (CMS), held at Gandhinagar in
February, 2020, a resolution (UNEP/CMS/Resolution 12.11 (Rev.
COP13) and Decision 13.46 were adopted inter-alia providing for
establishing, by COP14, under the umbrella of CMS an institutional
framework, under the leadership of India with the aim to agree on
conservation action for migratory birds.
y With a view to fulfill its commitment, India is organizing a two day
online meeting on 6th -7th October 2021 with CAF Range Countries,
anchored in Wildlife Institute of India.
y Approximately one in five of the world’s 11,000 bird species migrate,
some covering enormous distances.
y Conserving migratory birds requires cooperation and coordination
along the entire flyway between countries and across national
boundaries.

Limits:
It extends from the northernmost breeding grounds in the Russian
Federation (Siberia) to the southernmost non-breeding (wintering)
grounds in West and South Asia, the Maldives and British Indian Ocean
Territory.

Treaty:
Convention on Migratory Species (CMS) or the Bonn Convention: an
intergovernmental treaty (under the aegis of UNEP; signed in 1979
in Bonn, Germany, and entered into force in 1983) is the only global
convention specializing in the conservation of migratory species, their
habitats, and migration routes.

Major Species using this flyway:


Species such as the Baer’s pochard (CR); northern bald ibis (CR),
greater adjutant (EN) and black-necked crane (Vu), Indian skimmer
(Vu) etc.

In December 2020, The Union Ministry of Environment, Forest and


Climate Change released the “Status of Leopard in India, 2018” report.
14. Status of
Leopard in India Leopards:
2018
y Leopard (Panthera pardus) is the most widely distributed and
adaptable member of the family Felidae.

Environment 15
y The Indian subspecies, Panthera pardus fusca, is found in all
forested habitats in the country, absent only in the arid deserts
and above the timberline in the Himalayas (Prater 1980, Daniel
1996). In the Himalayas they are sympatric with snow leopards
(Panthera uncia) upto 5,200 m

Key takeaways from the Report:


y The leopard population has been estimated using camera trapping
method.
y There are 12,852 leopards in India as of 2018 as compared to the
previous estimate of 7910 conducted in 2014, an increase of 60%
in 4 years.
y The highest concentration of the leopard in India is estimated to
be in Madhya Pradesh (3,421) followed by Karnataka (1,783) and
Maharashtra (1,690).
y Recent meta-analyses of leopard status and distribution suggest
48–67% range loss for the species in Africa and 83–87% in Asia.
y In India, leopards have experienced a possibly human-induced 75-
90% population decline in the last ~120-200 years.
y In the Indian subcontinent, poaching, habitat loss, depletion of
natural prey and conflict are major threats to leopard populations.
y All these have resulted in changing the species status from ‘Near
Threatened’ to ‘Vulnerable’ by the IUCN.
y As for region-wise distribution, the highest number of 8,071
leopards were found in central India and eastern ghats.
y In the northeast hills, there are just 141 leopards.

Conservation Status:
y IUCN Red List: Vulnerable
y Wildlife (Protection)Act,1972: Schedule I
y CITES: Appendix I

Climate Change

In the news
y The southern bank of South Lhonak Lake in Sikkim breached,
releasing a powerful mixture of icy water and debris.
⚪ The resulting floods devastated the Chungthang Dam, NHPC
hydropower projects, and the region’s infrastructure.
15. Glacial lake
outburst flood
Glacial Lake Outburst Flood
(GLOF)
y Glacial lakes are large bodies of water that sit in front of, on top of,
or beneath a melting glacier. Also, Glacial lakes can be formed near
the snout of glaciers when meltwater accumulates.
⚪ Glacial lakes are mostly dammed by unstable ice or sediment
composed of loose rock and debris.

16 Environment
y Glacial lake outburst floods or GLOF occur when the boundary
around them breaks, huge amounts of water rush down the side
of the mountains, which could cause flooding in the downstream
areas.
⚪ GLOF can be triggered by several reasons, including earthquakes,
extremely heavy rains and ice avalanches.
Causes
y Climate Change and Glacier Melting: The increased warming of
the troposphere has led to the accelerated melting of glaciers in
various parts of the world. This increased glcaial melt releases
water, which can accumulate in depressions, forming lakes. These
glacial lakes are temporary and pose significant hazards.
y Glacier Retreat: As glaciers melt and retreat due to rising
temperatures, they leave behind depressions or basins. These
depressions can fill with meltwater and ice, forming glacial lakes.
When these lakes become too large, the pressure on the damming
material increases, increasing the risk of a GLOF.
y Glacier Surge: Some glaciers can experience surges, which are
periods of rapid advance and retreat. During a surge, a glacier can
dam up meltwater, creating a temporary glacial lake. When the
surge ends, the dam can breach, causing a GLOF.
y High Vulnerability Quotient: The embankments of these lakes
consist of loose deposits of glacier moraine, rocks, boulders, soil
and ice. Since these embankments are not properly compacted,
they have a high vulnerability quotient.
y Avalanche or Landslide: Avalanches, rockfalls, or landslides can
impact the damming material, causing it to weaken or collapse and
allowing the lake’s water to escape suddenly.

y Earthquakes: Seismic activity can trigger GLOFs by fracturing or


weakening the damming material. In some cases, earthquakes can
also dislodge ice and debris into the lake, leading to a rapid rise in
water levels and subsequent flooding.
y Volcanic Activity: Volcanic eruptions can lead to the melting of
glaciers and the release of massive amounts of water, which can
cause a GLOF.
y Human Activities: Human activities, such as mining, construction,
or deforestation in the vicinity of glacial lakes, can destabilize the
natural barriers and increase the risk of a GLOF.
⚪ Artificial Lake Formation: Artificial glacial lakes can form as
a result of construction projects, such as hydropower dams
or mining activities. Poorly designed infrastructure and
maintenance can lead to a higher risk of GLOFs.
Impact
y Loss of Life and Property: GLOFs can kill people, destroy houses,
bridges, roads, forests, and farmland, as well as livestock and
crops.

Environment 17
y Disruption of Livelihoods: GLOFs can affect the livelihoods of the
local communities for long periods, by reducing their access to
resources, markets, services, and opportunities. GLOFs can also
damage the tourism industry, which is a major source of income
for many mountain regions.
y Damage to Infrastructure and Environment: GLOFs can damage
or destroy hydropower plants, which are important for providing
electricity and reducing greenhouse gas emissions. GLOFs can also
alter the landscape, erode the soil, increase the sediment load in
the rivers, and affect the water quality and availability.
y Trans-boundary Impact: GLOFs can also affect the downstream
areas far from the glaciated headwaters where the threats originate.

NDMA Guidelines for Management of GLOFs


y Identifying Potentially Dangerous Lakes:
⚪ Recognizing potentially dangerous lakes involves a
multidisciplinary approach. Field observations, historical
records, and analyzing geomorphological and geotechnical
characteristics can help identify high-risk areas.
⚪ This information can be used to prioritize monitoring and risk-
reduction efforts.
y Use of Technology:
⚪ The use of Synthetic-Aperture Radar imagery can greatly enhance
early warning systems. This technology can detect changes in
water bodies, monitor glacier movements, and identify new lake
formations, especially during monsoon seasons.

⚪ Remote monitoring from space can help to track changes in


lake conditions over time.
y Channeling Potential Floods:
⚪ Managing lakes structurally is an important aspect of risk
reduction. Techniques such as controlled breaching, pumping
out water, and tunnel construction can help lower the water
volume, reducing the risk of a GLOF.
⚪ These methods should be well-planned and executed to
minimize downstream impacts.
y Uniform Codes for Construction Activity:
⚪ The development of uniform construction codes for
infrastructure and land use planning in GLOF-prone areas is
essential.
⚪ These codes should consider the geological and hydrological
risks and incorporate measures to mitigate these risks in
construction projects.

18 Environment
y Enhancing Early Warning Systems (EWS):
⚪ Early warning systems are critical for disaster preparedness.
Implementing sensor- and monitoring-based technical
systems for GLOF early warning is vital, as it can provide timely
information to at-risk communities.
y Training Local Manpower:
⚪ Local communities play a crucial role in disaster response and
preparedness as it helps in the rapid and effective response to
GLOFs.
⚪ These trained individuals can perform search and rescue
operations, assist in setting up emergency shelters, and
distribute relief supplies, functioning as the first line of defense
in disaster situations.
y Comprehensive Alarm Systems:
⚪ Modern communication technology, such as cell phones and
smartphones, can complement or replace traditional alarming
infrastructure.
⚪ These technologies can be utilized for mass notification
systems that can reach a wider audience and provide timely
alerts in case of impending disasters.

Other Recent GLOF Incidents in India?


y June 2013 - Uttarakhand had received an unusual amount of rainfall
leading to the melting of the Chorabari glacier and the eruption of
the Mandakini river.
y August 2014 - A glacial lake outburst flood hit the village of Gya in
Ladakh
y February 2021 - Chamoli district in Uttarakhand witnessed flash
floods which are suspected to have been caused by GLOFs.

y FFGS is aimed at helping disaster management teams and


governments make timely evacuation plans ahead of the actual
event of flooding
⚪ A dedicated FFGS centre will be established in New Delhi.

Flash Floods:
16. Flash flood y Flood of short duration with a relatively high peak discharge in
guidance system which the time interval between the observable causative event
(FFGS) and the flood is less than six hours between the occurrence of the
rainfall and peak flood.
⚪ The flood situation worsens in the presence of choked drainage
lines or encroachments obstructing the natural flow of water.
y Flash floods differ from river floods in their short time scales
and occurrence on small spatial scales, which makes flash flood
forecasting a different challenge from large-river flood forecasting.

Environment 19
Challenges:
y Forecasting: flash floods are very difficult as an event can occur
within three to six hours and the water run-off quantity is very
high.
y Increased frequency of occurrence: extreme rainfall events have
increased due to climate change and south Asia is highly prone to
flash floods.
y High mortality rate: Data suggest that across the world, about
5,000 people die annually due to flash floods.
y Lack of robust warning system: Despite such high mortality, there
was no robust forecasting or warning system for flash floods.

South Asian Flash Flood Guidance System:


y It has been developed by US-based Hydrologic Research Centre
after the World Meteorological Organisation (WMO) and few South
Asian countries put forth their views and the urgent need for such
a warning system.
⚪ Mitigate adverse impacts of hydrometeorological hazards;
⚪ Enhance NMHSs capacity to issue flash flood warnings and
alerts;
⚪ Enhance collaborations between NMHSs and DMAs;
⚪ Provide extensive training to the hydrometeorological
forecasters;

Warning Mechanism:
y Based on the rainfall and potential flooding scenario, flash flood
warnings will be issued to respective nations.
y Flash flood threat warning will be issued six hours in advance,
whereas flood risk warning will be issued 24 hours in advance.
Warnings about watershed level will be issued 12 hours in advance.

India’s Role:
y India is leading the delegation of nations, including Bhutan, Sri Lanka,
Bangladesh and Nepal, in sharing hydrological and meteorological
data towards preparing flash flood forecasts.
y India’s National Disaster Management Authority (NDMA) and the
Central Water Commission (CWC) partnered in developing the
system.

20 Environment
Coral Reefs are under high risk and need protection globally. They are
important as:
y Coral Reefs occupy less than 1% of the earth’s surface but provide
a home to nearly 25% of marine life
y They protect the shoreline from waves and storms by forming
barriers.
y In India, they are protected under Schedule I of Wildlife Protection
Act (WPA), 1972.

About Global Coral Reef Monitoring Network (GCRMN)


17. Coral Reef y It is an operational network of the International Coral
Protection Reef Initiative that aims to provide the best available scientific
information on the status and trends of coral reef ecosystems
for their conservation and management.

Threats
y Ocean Acidification
y Invasive Species such as snowflake coral
y Coral Bleaching
y Water pollution
y Seal level rise
y Unsustainable fishing practices such as blast fishing

Bharat Stage Emission Standards (BSES)


y Introduced in the year 2000.
y They are set by the Central Pollution Control Board under the
Ministry of Environment and Climate Change.
y Objective: To keep air pollutants emitted by the internal combustion
engine of vehicles under control.
y They are based on European (EURO) emission standards.

How do BS6 emission norms differ from BS4?


y Diesel Particulate Filter (DPF) and Selective Catalytic Reduction
18. Bharat Stage (SCR) are being introduced with the roll-out of Bharat Stage VI
Norms norms, which were not a part of Bharat Stage IV.
y Real Driving Emission (RDE) will be introduced in India for the first
time with the implementation of Bharat Stage VI emission norms.
It will measure a vehicle’s emission in real-time conditions against
laboratory conditions.
y Onboard Diagnostics (OD) has been made mandatory for all
vehicles.
y Sulphur and Nitrogen Oxide content: Sulphur traces in BS6 fuel are
five times lower (10 ppm) as compared to sulphur traces in BS4
fuel (50 ppm). Further, nitrogen oxide level for BS6-grade diesel
engines and petrol engines will be brought down by 70% and 25%,
respectively.

Environment 21
y BS VI can bring PM in diesel cars down by 80 per cent . The new
norms will bring down nitrogen oxides from diesel cars by 70 per
cent and in petrol cars by 25 per cent.

Climate Smart Cities Assessment Framework 2.0


The Ministry of Housing and Urban Affairs has launched the Climate
Smart Cities Assessment Framework (CSCAF) 2.0, along with the
‘Streets for People Challenge’ under the Smart Cities Mission.
y Objective: To provide a clear roadmap for cities towards combating
Climate Change while planning and implementing their actions,
19. Climate including investments.
Smart Cities y Framework: It has 28 indicators across five categories namely;
Assessment ⚪ Energy and Green Buildings,
Framework 2.0 ⚪ Urban Planning, Green Cover & Biodiversity,
⚪ Mobility and Air Quality,
⚪ Water Management and
⚪ Waste Management.
y Implementing Agency: The Climate Centre for Cities under
National Institute of Urban Affairs (NIUA) is supporting MoHUA in
implementation of CSCAF.

Carbon Pricing
y Carbon pricing is an instrument that captures the external costs of
greenhouse gas (GHG) emissions.
y It shifts the responsibility of paying for the damages of climate
change from the public to the GHG emission producers.
y Putting a price on carbon is widely seen as the most cost-effective
and flexible way to achieve emission reduction.

Benefits of Carbon Pricing


y Help facilitate emission pathways compatible with keeping global
20. Carbon Pricing temperature rise to below 2°C above pre-industrial levels.
y Spur investment and innovation in clean technology by increasing
the relative cost of using carbon-intensive technology.
y Promote the achievement of the Sustainable Development Goals
by channeling financing to sustainable development projects.
y Generate revenue which can be recycled into the green economy
through government spending for research and development into
green technology.
y Create environmental, health, economic, and social co-benefits,
ranging from public health benefits coming from reduced air
pollution to green job creation.

22 Environment
In a recent study, an Indian
researcher traced the tectonic
evolution and the nature of the
Greater Maldive Ridge (GMR).
y The study was conducted by the Indian Institute of Geomagnetism,
Mumbai, an autonomous institute of the Department of Science &
Technology, Govt. of India.

What is the Greater Maldive Ridge?


y The Maldive Ridge is an aseismic ridge that is not associated with
earthquake activities. This ridge, located in the western Indian
Ocean, southwest of India, is not well investigated.
y It is of paramount importance to gain knowledge on the structure
and geodynamics of aseismic ridges (as it provides valuable inputs
towards understanding the evolution of ocean basins).

What is the Study About?


y It has chalked the possible geological cross-sections along the
GMR for the first time with the help of satellite-derived high-
resolution gravity data.
y It provides the crustal architecture and the state of gravitational
equilibrium between Earth’s crust and mantle (isostasy) of the
Greater Maldive Ridge segment of the larger Chagos-Laccadive
21. Greater Maldives
Ridge (CLR) system.
Ridge
⚪ Isostasy is the rising or settling of a portion of the Earth’s
lithosphere that occurs when weight is removed or added in
order to maintain equilibrium between buoyancy forces that
push the lithosphere upward and gravity forces that pull the
lithosphere downward.
Their study, based mainly on the interpretation of gravity anomalies
(small differences in the pull of gravity caused by the lateral variations of
density within the subsurface) with broadband seismic and refraction
seismic data, provided for the first time a three-dimensional picture
of the variation of Moho along the Greater Maldive Ridge and the
adjoining ocean basins.

The depth to the boundary between the earth’s crust and the mantle or
the Mohorovicic discontinuity (Moho) over the GMR was systematically
mapped along with the finer variation of effective elastic thickness
(Te) at the place.
y The study found that Moho is deeper over the MR segment and
shallows southwards in the Deep Sea Channel region (DSC).

State of Gravitational Equilibrium


y Study provides the crustal architecture as well as the state of
gravitational equilibrium between Earth’s crust and mantle of the
GMR segment of the larger Chagos-Laccadive Ridge (CLR) system.

Environment 23
y It was found that Moho is deeper across Maldive Ridge (MR)
segment and shallows southwards in Deep-Sea Channel region
(DSC). However, effective elastic thickness values were lower over
MR compared in the DSC region.

Recently, Earth has been hit by a powerful Geomagnetic Storm,


having a severity grade of G4 according to the US National Ocean and
Atmospheric Administration (NOAA).
y Severity grade of G4, which is the second-highest grade possible,
can potentially cause widespread voltage control problems for
power grids. It can also cause protection systems to mistakenly
trip key electric assets of the grid.

What is a Geomagnetic Storm?


y A geomagnetic storm refers to the disruptions to the Earth’s
magnetic field caused by solar emissions.
y When a Coronal Mass Ejection (CME) or a high-speed solar stream
reaches our planet, it slams into the magnetosphere.
⚪ The Earth’s magnetosphere is created by its magnetic fields
and it usually protects us from the particles emitted by the
Sun.
y When a CME or a high-speed stream arrives at Earth, it peels
open the planet’s magnetosphere, kind of like an onion. This
allows energetic solar wind particles to stream down and hit our
atmosphere over the poles.
22. Geomagnetic y Solar weather events like this can also supercharge auroras,
Storm sometimes making them visible in places where they wouldn’t
have been otherwise.

Implications of Such a Storm


y Space Weather:
⚪ Not all solar flares reach Earth, but solar flares/storms, Solar
Energetic Particles (SEPs), high-speed solar winds, and CMEs
that come close can impact space weather in near-Earth space
and the upper atmosphere.

y Problems for Spacecraft Operations:


⚪ Solar storms can hit operations of space-dependent services
like Global Positioning Systems (GPS), radio, and satellite
communications. Aircraft flights and space exploration
programmes are vulnerable.
y Disturbances in the Magnetosphere:
⚪ It can potentially create disturbances in the magnetosphere,
the protective shield surrounding the Earth.
⚪ Astronauts on spacewalks face health risks from possible
exposure to solar radiation outside the Earth’s protective
atmosphere.

24 Environment
y Kashmir’s highly fertile alluvial soil deposits called ‘karewas’ are
being destroyed in the name of development, much to the peril of
local people.
About
y The word Karewa in Kashmiri dialect means, “elevated table-land.”
y This term was first used by Godwin-Austin (1859) and later by
Lydekker (1878) for an unconsolidated to semi – consolidated
sand-clay-conglomerate sequence.
y These sediments occur as terraces, plateaus and mounds and rest
over the Paleozoic-Mesozoic sediments of the Kashmir ‘basin’.
⚪ Since the Paleozoic and Mesozoic sequences form the basement
for the Karewa sediments, a short description of these is also
included for a regional overview
y Most of the cultivated fields in the Kashmir Valley are situated on
the Karewa sediments.
y Utility:
⚪ They are thick deposits of alluvial soil and sediments like
sandstone and mudstone.
⚪ This makes them ideal for cultivation of saffron, almonds,
apples and several other cash crops.
 Kashmir saffron, which received a Geographical Indication
(GI) tag in 2020 for its longer and thicker stigmas, deep-
red colour, high aroma and bitter flavour, is grown on these
23. Karewas
karewas.
⚪ The karewa sediments hold fossils and remnants of many
human civilisations and habitations and are also the most
fertile spots in the valley.
⚪ They are also being excavated to be used in construction.
Formation of Karewas
During the formation of the Pir Panjal range in the Pleistocene period,
the mountain ranges blocked the natural drainage in the region and
formed a lake of 5,000 sq km. Later, the water receded and it led to
the formation of Karewas in the valleys between the mountains.

Destruction of Karewa
y Karewas are being destroyed in the name of development, mainly
for clay mining. Between 1995 and 2005, Karewas were destroyed to
facilitate the construction of the 125-km-long Qazigund-Baramulla
railway line.
y Damodar Karewa in Budgam was razed to the ground to build
Srinagar airport. Last year, the Baramulla administration permitted
the excavation of Karewas to obtain clay for the construction of the
Srinagar ring road.
Destruction of Karewas will result in huge loss of archeological heritage,
accumulation of silt in rivers like Jhelum which eventually leads to
floods. Thus, it is important to protect Karewas.

Environment 25
Recently, a study investigated why glaciers in the Karakoram Range of
Central-South Asia have not been as affected by Climate Change as
others.
y They have attributed this phenomenon called Karakoram Anomaly
to the recent revival of Western Disturbances (WDs).

What is Karakoram Anomaly?


y The ‘Karakoram Anomaly’ is termed as the stability or anomalous
growth of glaciers in the central Karakoram, in contrast to the
retreat of glaciers in other nearby mountainous ranges of Himalayas
and other mountainous ranges of the world.

Key Findings of the study


y It is for the first time that a study brought forth the importance
that enhanced western Disturbance (WD)-precipitation input
during the accumulation period plays in modulating regional
climatic anomalies.
24. Karakoram ⚪ Previous studies have highlighted the role of temperature in
Anomaly establishing and sustaining the anomaly over the years
y Western Disturbances (WDs) are the primary feeder of snowfall for
the region during winters.
y The study suggests they constitute about around 65% of the total
seasonal snowfall volume and about 53% of the total seasonal
precipitation, easily making them the most important source of
moisture.
y Further, the precipitation intensity of WDs impacting Karakoram
has increased by around 10% in the last two decades, which only
enhances their role in sustaining the regional anomaly.
Karakoram Range
y The Karakorams are part of a complex of mountain ranges at the
centre of Asia, including the HinduKush to the west, the Pamirs
to the northwest, the Kunlun Mountains to the northeast, and the
Himalayas to the southeast.
y The Karakorams cover parts of Afghanistan, China, India, Pakistan,
and Tajikistan.

Two Indian beaches, Minicoy Thundi beach and Kadmat beach, located
in Lakshadweep, have received the International eco-label ‘Blue Flag’.

About:
y With the new additions, the number of beaches certified under the
25. Blue Flag
Blue Flag Certification is twelve.
Certification
y The other Indian beaches in the blue list are Shivrajpur-Gujarat,
Ghoghla-Diu, Kasarkod and Padubidri-Karnataka, Kappad-Kerala,
Rushikonda- Andhra Pradesh, Golden-Odisha, Radhanagar-
Andaman and Nicobar, Kovalam in Tamil Nadu and Eden in
Puducherry beaches.

26 Environment
Blue Flag’ certification:
y The Blue Flag programme was started in France in 1985 and in
areas out of Europe in 2001. It promotes sustainable development
in freshwater and marine areas through four main criteria: water
quality, environmental management, environmental education and
safety.
y It can be obtained by a beach, marina, or sustainable boating
tourism operator, and serves as an eco-label.
y The certification is awarded by the Denmark-based non-profit
Foundation for Environmental Education (FEE). It is awarded
annually to beaches and marinas in FEE member countries.
y Forty-eight countries currently participate in the program, and
5042 beaches, marinas, and boats have this certification
Beach Environment & Aesthetics Management Services
y Beach Environment & Aesthetics Management Services comes
under ICZM (Integrated Coastal Zone Management) project.
y This was launched by the Society of Integrated Coastal Management
(SICOM) and the Union Ministry of Environment, Forest and Climate
Change (MoEFCC).
y The objectives of BEAMS program are to:
⚪ Abate pollution in coastal waters,
⚪ Promote sustainable development of beach facilities,
⚪ Protect & conserve coastal ecosystems & natural resources,
⚪ Strive and maintain high standards of cleanliness,
⚪ Hygiene & safety for beachgoers in accordance with coastal
environment & regulations.

Environmental Pollution

y The concentration of ammonia in the Yamuna river was 7.4 ppm


(parts per million)
y Acceptable limit
⚪ The acceptable maximum limit of ammonia in drinking water,
26. Ammonia as per the Bureau of Indian Standards, is 0.5 ppm
pollution y Toxicity
⚪ If the concentration of ammonia in water is above 1 ppm it is
toxic to fishes.
⚪ In humans, long term ingestion of water having ammonia levels
of 1 ppm or above may cause damage to internal organs.

y National Mission for Clean Ganga hosted a session on Liquid ‘Zero


Liquid Discharge Cities’.

27. Zero liquid About Zero-liquid discharge (ZLD)


discharge (ZLD)
y ZLD is a water treatment process in which all wastewater is purified
and recycled; therefore, leaving zero discharge at the end of the
treatment cycle.

Environment 27
⚪ The focus of ZLD is to reduce wastewater economically and
produce clean water that is suitable for reuse.
⚪ It employs advanced wastewater/desalination treatment
technologies to purify and recycle virtually all of the
wastewater produced.

y Microplastics are tiny plastic particles Less than 5mm in size


that result from both commercial product development and the
breakdown of larger plastics.
⚪ As a pollutant, microplastics can be harmful to the environment
and animal health.
⚪ Include microfibres (most abundant), fragments, pellets, flakes,
sheets or foams.
y They Enter water bodies through atmospheric deposition, run-
28. Microplastics
off from
y Microplastics larger than 150 micrometres are not likely to be
absorbed by the human body but the chance of absorbing very
small microplastic particles, including nano-sized plastics, are
higher.contaminated land or through municipal wastewater.
y They are different from Microbeads which are intentionally designed
to be small. They are used in many health and beauty products.
They pass unchanged through waterways into the ocean.

E-Waste
y E-waste is any electrical or electronic equipment that’s been
discarded. E-waste is particularly dangerous due to toxic chemicals
that naturally leach from the metals inside when buried.
y Government of India notified E-Waste (Management) Rules, 2022
with a major aim to digitize the e-waste management process and
enhance visibility.

E-Waste Management Rules 2022


y Registration of stakeholders: Entities like manufacturer, producer,
29. E-Waste recycler, refurbisher shall be required to apply for EPR registration.
Management Further, these entities are not allowed to deal with unregistered
manufacturers, producers, recyclers and refurbishers.
y Applicability: The products on which e-waste management rules
apply can be bifurcated into two categories as follows:
⚪ Consumer Electricals and Electronics
⚪ IT and Telecommunication Equipment
y Storage of E-waste: Any manufacturer, producer, recycler or
refurbisher can store the e-waste only for a period of up to 180 days
are required to maintain the record of its sale, transfer or storage.
⚪ The CPCB has the power to extend storage period up to 365
days specifically for recycling or reuse of e-waste.

28 Environment
y Reduction in use of Hazardous Substances: The producers of
e-waste have been specified proportions of hazardous substances
in their new products.
⚪ Examples of hazardous substances include Mercury, Lead,
Cadmium, Polybrominated Biphenyls and Hexavalent Chromium,
Polybrominated Diphenyl Ethers.
y Digitized systems approach: The EPR obligations shall be discharged
by producers through the online purchase of certificates from
registered recyclers.
⚪ Standardizing the e-waste value chain through a common
digital ‘portal’ may ensure transparency.
⚪ It is also crucial to reduce the frequency of ‘paper trading’ or
‘false trail’, i.e., a practice of falsely revealing 100% collection on
paper while collecting and/or weighing ‘scrap’ to meet targets.
y The e-waste handling should also inculcate best practices to
incorporate the informal sector in an organized e-waste network.
y For instance, ‘Karo Sambhav’, a Delhi-based PRO, has integrated
informal aggregators in its collection mechanism.
y Through this initiative, e-waste is entered in a safe and structured
system and the informal sector also has an advantage in terms of
financial and legal security.

Summit, Conferences, and Conventions

International Convention on Civil Liability for Bunker Oil Pollution


Damage, also known as “The Bunker Convention” was adopted to
ensure that adequate, prompt, and effective compensation is available
to persons who suffer damage caused by spills of oil, when carried as
fuel in ships’ bunkers.

Compensation under the Convention


y Rights to compensation shall be extinguished unless an action is
brought thereunder within three years from the date when the
damage occurred.
30. Bunker
y However, in no case shall an action be brought more than six years
Convention
from the date of the incident which caused the damage.
y There is a provision of requirement for direct action - this would
allow a claim for compensation for pollution damage to be brought
directly against an insurer.

Oil Spills
y An oil spill refers to any uncontrolled release of crude oil, gasoline,
fuels, or other oil by-products into the environment.
y Oil spills can pollute land, air, or water, though it is mostly used for
oceanic oil spills.

Environment 29
Causes of oil spill:
y Intensified petroleum exploration and production on continental
shelves and the transport of large amounts of oils in vessels.
y Oil spills that happen in rivers, bays and the ocean most often are
caused by accidents involving tankers, barges, pipelines, refineries,
drilling rigs and storage facilities, but also occur from recreational
boats and natural disasters.

Environmental Impact
y Threat to indigenous people who depend on seafood
y Harmful to aquatic life as it prevents sufficient amounts of sunlight
from penetrating the surface, and it also reduces the level of
dissolved oxygen.
y Hypothermia as it ruins the insulating and waterproofing properties
of feathers and fur of birds
y Toxicity in affected animals

y SASCOF is a consortium of meteorologists and hydrological experts


from South Asian countries supported by the World Meteorological
Organization (WMO).
y Member countries: Afghanistan, Pakistan, India, Nepal, Bangladesh,
Sri Lanka, Bhutan, Maldives and Myanmar.
31. South Asian y Objective: To prepare consensus seasonal climate information
Climate Outlook on a regional scale that provides a consistent basis for preparing
Forum (SASCOF) national-level outlooks
⚪ Such platforms also serve to interact with the user sector to
understand and enhance the use of climate information.
y They work collectively to issue regional forecasts and the team
releases forecasts for the Southwest and Northeast monsoon
seasons, every year.

y Global Platform for Disaster Risk Reduction, 2022 (GP DRR 2022)
took place in Indonesia.
y The outcome of GP2022 was summarized in the co-chairs’ Bali
Agenda for Resilience.
y Theme: From Risk to Resilience: Towards Sustainable Development
For All in a Covid-19 Transformed World.
32. Global Platform
for Disaster Risk Important outcomes
Reduction 2022 y There is a need for a whole-of-society approach to Disaster Risk
(GP2022) Reduction (DRR), ensuring no one is left behind
y DRR must be at the core of development and finance policies,
legislation and plans to achieve the 2030 Agenda for Sustainable
Development and the Sustainable Development Goals (SDG).
y Current greenhouse gas emission levels far exceed their mitigation,
resulting in an increase in frequency and intensity of catastrophic
events.

30 Environment
y DRR and climate change adaptation have the common objective of
reducing vulnerability and enhancing capacity as well as resilience.

Sendai Framework for Disaster Risk Reduction 2015-2030


y The Sendai Framework for Disaster Risk Reduction 2015-2030 was
adopted at the Third UN World Conference in Sendai, Japan, on
March 18, 2015.
y It is the successor agreement to the Hyogo Framework for Action
(2005–2015), which was the most encompassing international
accord to date on disaster risk reduction.
y It applies to the risk of small-scale and large-scale, frequent and
infrequent, sudden and slow-onset disasters caused by natural or
man-made hazards, as well as related environmental, technological
and biological hazards and risks.
y The Sendai Framework sets four specific priorities for action:
⚪ Understanding disaster risk;
⚪ Strengthening disaster risk governance to manage disaster risk;
⚪ Investing in disaster risk reduction for resilience;
⚪ Enhancing disaster preparedness for effective response,
and to “Building Back Better” in recovery, rehabilitation and
reconstruction.

y The International Solar Alliance is a member-driven, action-


oriented platform that increases the deployment of solar energy
technologies. It aims to bring energy access, drive energy transition,
and ensure energy security.
y The ISA initiative proposed by India and France at the COP 15 in
Paris.
y The countries that are solar-rich, that lie between the Tropic of
Cancer and Tropic of Capricorn are members of the International
Solar Alliance. They are able to develop the low carbon growth
path offered by solar power, with particular emphasis on least
developed countries.
33. International
y Working:
Solar Alliance
⚪ It facilitates the sharing of knowledge and expertise among
(ISA)
member countries,
⚪ It encourages research and development in solar technology,
and
⚪ It promotes capacity building and skill development in the solar
sector.

In news recently
The highlights of recent ISA conference:
y Increase in Viability Gap Funding (VGF) for projects, raising it to a
range of 10% to 35% to foster greater investments, particularly in
African countries.

Environment 31
y Four projects supported by the ISA were inaugurated during the
assembly. These initiatives spanned across different countries:
⚪ Solarization of the Malawi Parliament
⚪ Solarization of rural healthcare centers in Fiji
⚪ Installation of a solar-powered cold storage in Seychelles
⚪ Solarization of a school in Kiribati
y Renewable energy could potentially supply 65% of the world’s total
electricity by 2030 and decarbonize 90% of the power sector by
2050.

Ease of Doing Solar Report 2020


y A report entitled Doing Solar Report 2020 was released by the
International Solar Alliance on 24th February 2021. Furthermore,
ISA launched the World Solar Bank at UNFCCC COP 26 in 2021.
Also, International Solar Alliance has re-elected India and France
as its President and Secretary-General respectively for the term
of 2 years at a virtual assembly of the International Solar Alliance.
World Solar Bank
y The World Solar Bank was launched by ISA at the Glasgow
Conference in 2021.
y Objective: To transfer the solar power generated in an electricity
region to meet the requirements of other regions

St. Petersburg Declaration


y Countries pledged to double the number of tigers by 2022 (Chinese
Year of the Tiger) under Petersburg Declaration.
y India has already achieved the target of doubling the count.
y The Declaration recognized that Asia’s most iconic animal faces
imminent extinction in the wild

Objective
y Strive to double the number of wild tigers across their range by
2022
34. St. Petersburg
declaration Declaration
y To effectively manage, preserve, protect, and enhance habitats
y Work collaboratively to eradicate poaching, smuggling, and illegal
trade of tigers, their parts, and derivatives
y Engage with indigenous and local communities to gain their
participation in biodiversity conservation, minimize negative
impacts on tigers, their prey, and habitats, and reduce the incidence
of human-tiger conflict by providing sustainable and alternative
livelihood options through financial support, technical guidance,
and other measures.
y Increase the effectiveness of tiger and habitat management

32 Environment
y Appealing for the commitment of international financial
institutions, such as World Bank, Global Environment Facility, Asian
Development Bank, bilateral and other donors and foundations,
CITES Secretariat, etc.

The UN Water Conference 2023


y It was held in New York (the first such meeting on water after 46
years). Also, a mid-term review of the UN Decade for Action on
Water and Sanitation (2018-2028) was held at the conference.
y It was co-hosted by the Netherlands and Tajikistan.
y Objective – To raise awareness of the global water crisis and decide
on action to achieve internationally agreed water-related goals.
Themes: The conference was based on the following five themes
y Water for Health: Access to ‘WASH’ (Global Water, Sanitation, &
Hygiene) including the Human Rights to Safe Drinking Water and
Sanitation.
y Water for Sustainable Development: Valuing water, the water-
energy-food nexus and sustainable economic and urban
development.
y Water for Climate, Resilience and Environment: Source to sea,
biodiversity, climate, resilience and disaster risk reduction.
y Water for Cooperation: Transboundary and international water
cooperation, cross sectoral cooperation and water across the 2030
Agenda.
35. The UN Water y Water Action Decade: Accelerating the implementation of the
Conference objectives of the Decade for Action, including through the UN
Secretary-General’s Action Plan.

Outcome of the Water Conference


y Water Action Agenda : 700 voluntary commitments to form the
Water Action Agenda.
y Climate resilient water and sanitation infrastructure – USA
announced a commitment of up to $49 billion in investments to
support climate resilient water and sanitation infrastructure and
services
y Quality Infrastructure – Japan announced that it will contribute
500 billion yen to the solution of water-related social issues faced
by the Asia-Pacific region by developing quality Infrastructure
y River basins management and clean running water –Vietnam
pledged to develop policies for major river basins management by
2025 and clean running water by 2030

y Africa’s water investments gap – The African Union Commission


and Continental Africa Investment Programme (AIP) aims to close
Africa’s water investments gap by mobilising at least $30 billion
per year by 2030.

Environment 33
y European Union (EU) – The EU aims to support 70 million individuals
to an improved drinking water source and sanitation facility by 2030.
y Water Convention and transboundary cooperation – Switzerland
submitted 5 commitments in the areas of Water Convention and
transboundary cooperation.
Important Note: The first UN Water Conference was held in Argentina
in 1977. It resulted in the first global ‘Action Plan’ recognizing that
“all peoples, whatever their stage of development and social and
economic conditions, have the right to have access to drinking water
in quantities and of a quality equal to their basic needs.”

CPLC is a voluntary initiative that brings together leaders from


government, business, civil society and academia to enhance global
understanding of carbon pricing as a tool for accelerating and financing
effective climate action.

Carbon Pricing Leadership Report: Key Findings


y To meet the Paris Agreement temperature goals less than 4% of
global emissions are currently covered by a carbon price within the
range needed by 2030.
y To meet the Paris Agreement target several countries increased
their carbon tax rates and adopted more ambitious trajectories,
and pilot ETS are being considered in a number of countries.
y Agreement on Article 6 is encouraging further interest in carbon
markets.
y It gives countries the tools for environmental integrity to avoid
double counting and paves the way to get private capital flowing
to developing countries.
36. Carbon Pricing
Leadership Key Messages
Coalition
y Net zero involves first cutting emissions to a minimum, then
seeking (permanent) removals.
y The world is not on track to achieve net zero by 2050.
y Carbon pricing is an important mitigation tool, but it isn’t a silver
bullet.
y Carbon markets are intended as a tool for climate action and to
leverage investment.
y High-quality carbon credits are crucial for long-term emissions
reduction.
y Corporate net zero commitments should include value chain
emissions.
y Net zero should be integrated into all investment decisions.
y The path to net zero should be socially fair and just.
y The credibility of ambition and stakeholder engagement in net
zero development and implementation processes depend on
transparent net zero targets.
y Transparency, clear targets, and accountability are key.

34 Environment
In Oct 2021, India joined the High Ambition Coalition (HAC) for Nature
and People.
y India is the first of the BRICS (Brazil, Russia, India, China and South
Africa) to join the HAC.
y India’s announcement comes in the lead up to a high-level
biodiversity meeting, hosted by China.

High Ambition Coalition for Nature and People


y High Ambition Coalition for Nature and People, was initiated at the
“One Planet Summit” in Paris in January 2021.
37. High Ambition
y It is co-chaired by Costa Rica and France and by the United
Coalition for
Kingdom as Ocean co-chair.
Nature and
y It is a group of more than 70 countries encouraging the adoption of
People
the global goal to protect 30×30.
y HAC champions a global deal for nature and people with the central
goal of protecting at least 30 percent of world’s land and ocean by
2030.
y The 30×30 target is a global target which aims to halt the
accelerating loss of species, and protect vital ecosystems that are
the source of our economic security.
y HAC members currently include a mix of countries in the global
north and south; European, Latin American, Africa and Asia
countries are among the members.

Recently, the UN Convention to Combat Desertification (UNCCD) has


announced the launch of its first-ever Data Dashboard, which shows
that Land Degradation is advancing at an astonishing rate across all
regions.
y It compiled national reporting figures from 126 countries to provide
a comprehensive overview of the Land Degradation situation
globally.
y The 21st session of the UNCCD will take place in Samarkand,
Uzbekistan, in November 2023. It will focus on reviewing global
38. UNCCD: Land
progress towards achieving Land Degradation Neutrality (LDN) and
Degradation
addressing related issues.
Neutrality Fund
Land Degradation Neutrality (LDN)
y LDN is a simple idea and a powerful tool, means to secure enough
healthy and productive natural resources by avoiding Degradation
whenever possible and restoring land that has already been
degraded.
y At its core are better land management practices and better land-
use planning that will improve economic, social and ecological
sustainability for present and future generations.

Environment 35
y LDN provides significant benefits in terms of mitigation and
adaptation to climate change. Halting and reversing land degradation
can transform the land from being a source of Greenhouse Gas
Emissions (GHG) to a carbon sink, by increasing carbon stocks in
soils and vegetation.

Key Highlights
y Land Degradation Trends:
⚪ From 2015 to 2019, the world lost over 100 million hectares of
productive land annually, which is twice the size of Greenland.
⚪ Land degradation is worsening rapidly on a global scale.
y Regional Variances:
⚪ Eastern and Central Asia, Latin America, and the Caribbean
experience severe degradation, impacting at least 20% of their
total land area.
⚪ Sub-Saharan Africa, Western and Southern Asia, Latin America,
and the Caribbean have experienced land degradation rates
faster than the global average.
⚪ In sub-Saharan Africa and in Latin America and the Caribbean,
163 million hectares and 108 million hectares, respectively, have
succumbed to land degradation since 2015.

y Bright Spots:
⚪ Some countries have shown progress in combating land
degradation. For instance, In sub-Saharan Africa, Botswana
reduced land degradation from 36% to 17% of its territory.
 The country has committed a total of 45.3 million
hectares to LDN, including both measures to avoid further
degradation as well as restoration interventions in selected
land degradation hotspots.
⚪ In the Dominican Republic, the proportion of degraded land
has decreased from 49% to 31% between 2015 and 2019,
with ongoing efforts to restore 240 000 hectares in the Yaque
del Norte River basin and in cocoa production areas in San
Francisco de Macoris province.
⚪ While Uzbekistan reported the highest proportion of degraded
land (26.1%) in the Central Asia region, it also saw the largest
decrease – from 30% to 26% compared to 2015.
 Between 2018-2022, Uzbekistan carried out saxaul planting
to eliminate salt and dust emissions from the drained
bottom of the Aral Sea.
y India’s Statistics:
⚪ Degraded Land Area in India has been increased from 4.42% in
2015 to 9.45 % in 2019.

36 Environment
Recommendations of UNCCD to Achieve LDN Targets:
y UNCCD data emphasizes the need to restore 1.5 billion hectares
of degraded land by 2030 to achieve LDN targets outlined in the
United Nations Sustainable Development Goals.
y The UNCCD highlights that although global trends are concerning,
it is still possible to meet or exceed LDN goals through stopping
further degradation and accelerating restoration efforts.
y Many countries have set voluntary LDN targets for 2030, and
funding is crucial for these efforts.

Recently, the 5th International Marine Protected Areas Congress


(IMPAC5) was held in Canada in order to discuss the solutions to
address the Funding Gap of Marine Protected Areas (MPAs).
y This meeting is crucial since nations agreed to protect 30% of
Earth’s lands and oceans by 2030 at the 15th Conference of the
Parties to the Convention on Biological Diversity held in 2022.

Highlights of the Meet


y Sustainable and Resilient MPA networks:
⚪ As many as 70% of MPAs are underfunded. A well-managed and
sufficiently funded MPA can restore good health to vulnerable
ecosystems.
⚪ Achieving sustainable and resilient MPA networks depends on
an overall commitment to protection, leadership, engagement
39. 5th International from stakeholders, institutions, governments and organizations,
Marine Protected Indigenous peoples, coastal communities, and individuals in an
Areas Congress inclusive and equitable manner to advance ocean protection.
⚪ IMPAC5 aims to provide a forum for sharing knowledge,
successes and best practices in an open and respectful
environment for the exchanging of ideas among a diversity of
views.

What are Marine Protected Areas?


y MPAs are designated areas of the ocean that are set aside for
the protection and conservation of marine ecosystems and their
biodiversity.
y Within the region, certain activities are limited, or entirely prohibited,
to meet specific conservation, habitat protection, ecosystem
monitoring or fisheries management objectives.
y MPAs do not necessarily exclude fishing, research or other human
activities; in fact, many MPAs are multi-purpose areas.

Environment 37
Recently, the UN (United Nations) Ocean Conference 2022 was held to
ensure global cooperation towards protection and sustenance of the
Ocean ecosystem of the world.
y The conference was co-hosted by the Governments of Kenya and
Portugal.
y The UN Ocean conference 2022 is aligned to SDG (Sustainable
Development Goals) 14 ‘life below water’ and stresses on the
critical need for scientific knowledge and marine technology to
build ocean resilience.

Initiatives to Ensure Sustainable Ocean Ecosystem


y Decade of Ocean Science for Sustainable Development:
⚪ The UN has proclaimed a Decade of Ocean Science for
Sustainable Development (2021-2030) to support efforts to
reverse the cycle of decline in ocean health and gather ocean
40. UN Ocean
stakeholders worldwide behind a common framework.
Conference
y World Oceans Day:
⚪ 8th June is World Oceans Day, the United Nations day for
celebrating the role of the oceans in our everyday life and
inspiring action to protect the ocean and sustainably use
marine resources.
y Marine Protected Areas:
⚪ In general terms, a Marine Protected Area (MPA) is a marine area
that provides protection for all or part of the natural resources
it contains.
y GloLitter Partnerships Project:
⚪ It is launched by the International Maritime Organization (IMO)
and the Food and Agriculture Organization of the UNs (FAO)
and initial funding from the Government of Norway. It is aimed
to prevent and reduce marine plastic litter from shipping and
fisheries.

y The 1972 United Nations Conference on the Environment’ or ‘The


Stockholm conference’ recently completed its 50 years.
y To commemorate this, the Stockholm +50 conference is scheduled
to be held in Sweden.
y This is being held at a time when the world is facing a triple
41. Stockholm planetary crisis of climate change, pollution and waste, nature
Conference, 1972 and biodiversity loss, as well as other planetary issues even after
50 years of the Stockholm Declaration. This is threatening the
achievement of the Sustainable Development Goals.
Background:
y Climate change was first discussed using emerging scientific
evidence in the UN General Assembly in 1968.

38 Environment
⚪ In 1967, a research study provided the actual estimates of
global temperature based on CO2 levels. Also, it was predicted
that the doubling of CO2 from the current level would result in
nearly 2°C rise in global temperature.
y The idea of the Stockholm Conference was first proposed by
Sweden. That’s why it’s also termed the “Swedish Initiative”.

About:
y The United Nations Conference on the Human Environment in
Stockholm was held from 5th –16th June, 1972.
y This was the first global convergence on the planetary environment.
y The theme was ‘Only One Earth’.

Stockholm Declaration and Action Plan for the Human Environment


y Stockholm Declaration:
⚪ The Stockholm Declaration contained 26 principles that marked
the beginning of a dialogue between developed and developing
countries.
⚪ This built the “interconnections between development, poverty
and the environment”.
y Action Plan:
⚪ The Action Plan contained three main categories that were
further broken down into 109 recommendations:
 Global Environmental Assessment Programme (watch plan)
 Environmental management activities
 International measures to support assessment and
management activities carried out at the national and
international levels.

Three Dimension of the Conference


y Countries agreed to not “harm each other’s environment or the
areas beyond national jurisdiction”.
y An action plan to study the threat to Earth’s environment.
y Establishment of an international body called the UN Environment
programme (UNEP) to bring in cooperation among countries.

Key agreements of the Stockholm Declaration


y Natural resources such as air, water, land, flora and fauna must be
safeguarded through careful planning for the benefit of the present
and future generations.
y The release of toxic substances and heat emissions should not be
allowed to exceed the capacity of the environment.
y The poor and developing nations must be supported in their
struggle against pollution.
y The environmental policies of the states should support the
present or future development potential of developing countries.

Environment 39
y Appropriate steps should be taken by States and international
organisations to reach an agreement on meeting the possible
national and international economic consequences resulting from
the application of environmental measures.
y According to the UN charter and principles of international law, the
States have the sovereign right to exploit their own resources as
per their own environmental policies.
⚪ However, the states have this responsibility of making sure that
activities within their jurisdiction or control do not cause any
harm to the environment of other States or of areas beyond
the limits of national jurisdiction.

y Recently, India pushed for a new fund to reverse biodiversity loss


at the U.N. biodiversity conference (COP-15) in Canada’s Montreal.

Key Points of CBD CoP 15


y Global Biodiversity Framework (GBF):
⚪ The draft GBF, set to replace the Aichi Biodiversity Targets,
comprises 22 targets and four goals proposed for 2030—a
stepping stone to the 2050 goal of Living in Harmony with
Nature.
⚪ The GBF targets include reducing pollution, pesticides, subsidies
harmful to nature and the rate of introduction of invasive alien
species among others.
⚪ It will cater to a new set of goals and targets to halt and reverse
biodiversity loss.
42. Convention
⚪ As the 196 parties to the Convention on Biological Diversity
on Biological
(CBD) finalise negotiations for a post-2020 GBF, there have
Diversity (CBD)
been calls for the inclusion of the CBDR principle in finance-
related targets.
y Fund at present:
⚪ Presently, the Global Environment Facility caters to multiple
conventions, including the UNFCCC and UN Convention to
Combat Desertification.
⚪ It remains the only source of funding for biodiversity conservation
right now.
y Subsidy elimination:
⚪ Eliminating subsidies that are harmful to the environment,
such as subsidies for fossil fuel production, agriculture, forestry
and fisheries, by at least $500 billion (one billion = 100 crore)
annually.
⚪ This money will be used for biodiversity conservation.

40 Environment
y Target:
⚪ It aims at achieving a historic deal to halt and reverse biodiversity
loss on par with the 2015 Paris Agreement on climate change,
when all countries agreed to holding the increase in the global
average temperature to well below 2 degrees Celsius above the
pre-industrial level.

CBDR
y It was established as the seventh principle of the Rio Declaration
adopted at the Earth Summit in 1992.
y CBDR is defined as “states have common but differentiated
responsibilities in view of the different contributions to global
environmental degradation”.
y Applying the CBDR principle to biodiversity conservation has not
been straightforward as compared to climate negotiations, and
there have been repeated disagreements between the global north
and south on the issue.
y The CBD is known informally as the Biodiversity Convention.
y It is a multilateral treaty.
y The convention was opened for signature at the Earth Summit in
Rio de Janeiro in 1992 and entered into force in 1993.
y The Secretariat of the Convention on Biological Diversity (SCBD) is
based in Montreal, Canada.
y Two supplementary agreements:
⚪ The Cartagena Protocol on Biosafety to the Convention on
Biological Diversity is an international treaty governing the
movements of living modified organisms (LMOs) resulting from
modern biotechnology from one country to another.
⚪ The Nagoya Protocol on Access to Genetic Resources and
the Fair and Equitable Sharing of Benefits Arising from their
Utilization (ABS) to the Convention on Biological Diversity is
another supplementary agreement to the CBD.
y The CBD’s governing body is the Conference of the Parties (COP).
⚪ This ultimate authority of all governments (or Parties) that have
ratified the treaty meets every two years to review progress, set
priorities and commit to work plans.

During the COP28 Local Climate Action Summit, Bloomberg


Philanthropies, Clean Air Fund, and C40 Cities announced a new cohort
of cities to benefit from funding, technical support, air quality data,
community engagement, capacity building, and additional support as
part of the Breathe Cities clean air initiative.
43. Clean Air Fund
y Clean Air Fund is a global philanthropic organisation working with
governments, funders, businesses and campaigners to create a
future where everyone breathes clean air. Clean Air Fund provides
funds and partners with organisations that promote air quality
data, build public demand for clean air and drive policy change.

Environment 41
Organizational Structure:
y The Clean Air Fund is a UK registered charity governed by a Board
of Trustees. Our Trustees have legal and financial responsibility
for our activities and delegate the day-to-day running of the
organisation to our Senior Leadership Team.
y Geographical Coverage : Global

Objectives:
y CAF is raising pollution up the political agenda through partnerships
with the World Economic Forum, the World Health Organization
and the C40 Cities’ global Mayors forum.

Activities:
y CAF call on decision makers to:
y Stop investing in air polluting fossil fuels, Commit new funding to
support the transition to clean energy solutions, Make action on
air pollution an explicit priority in climate action and sustainable
development activities.

y 5th International Maritime Protected Areas Congress (IMPAC) took


place in Vancouver, Canada in February 2023. As part of achieving
its goals, the IMPAC is examining, among other options, Other
Effective Area-Based Conservation Measures as well.
y On World Wetlands Day, that is, on February 2, the Aravalli
Biodiversity Park was announced as the first Other Effective Area
– based Conservation Measures site, OECM site.

OECM Meaning:
y Other effective area-based conservation measures’ (OECMs) are
areas that aim to achieve the long-term and effective in-situ
conservation of biodiversity outside of protected areas.
y An OECM is a geographically defined area other than a Protected
44. OECM Status Area, which is governed and managed in ways that achieve positive
and sustained long-term outcomes for the in-situ conservation of
biodiversity, with associated ecosystem functions and services and
where applicable, cultural, spiritual, socio-economic, and other
locally relevant values.

Simple layman terms:


y “OECM is not a protected area. It is governed to achieve positive
outcomes. The outcomes are conservation of biodiversity, giving
importance to cultural, socio – economic and spiritual values.”

Background:
y In 2022, 196 countries agreed to safeguard 30% of the world’s
terrestrial, inland, and marine resources by 2030.

42 Environment
y In 2022, the Kunming-Montreal Global Biodiversity Framework was
accepted at the 15th Conference of Parties (COP15) to the UN
Convention on Biological Diversity (CBD) in Canada’s Montreal.

About the Aravalli Biodiversity Park:


y It is spread over 390 acres. It has a semi – arid vegetation. It has
more than 43,000 shrubs, 101,000 trees and 300 endemic plant
species. Earlier, the park was a mining site.
y It was transformed into a city forest through immense efforts
of environmentalists, scientists, ecologists along with the local
population. Aravallis are considered as the green lungs of Delhi.
y They support leopards, foxes, sambhars and jackals. The Aravallis
provide 7.07% of oxygen to Delhi.

The Clydebank Declaration was signed at the COP26 Climate summit


held in Glasgow in November 2021. Under the declaration, the
signatories agreed to work together to support the establishment of
green shipping corridors.
About:
y The signatories of the Clydebank Declaration will establish zero
– emission maritime routes between two or more routes. These
routes are called green shipping corridors.
y The objective is to establish six green corridors by the middle of
the decade, that is by 2025.
y Under the declaration, more than two hundred businesses have
committed to increase and commercialize zero emission fuels and
vessels by 2030.
y Also, the buyers such as Ikea, Amazon, Patagonia, Michelin have
already announced that they will buy only zero carbon freight from
45. Clydebank 2040.
Declaration y It was signed by Australia, Belgium, Chile, Costa Rica, Fiji, Denmark,
Finland, Fiji, France, Republic of Ireland, Germany, Japan Italy,
Morocco, Republic of Marshall Islands, Netherlands, Spain, Norway,
Sweden, USA and UK.
y The declaration was named as “Clydebank” to pay tribute to the
heritage city Clydebank and to the River Clyde.
y The Antonine Wall is in Clydebank. It was declared a World Heritage
Site in 2008.
y The wall was a turf fortification. It was built by the Romans. Also,
Clydebank is one of the major ship building centre.

Pledges:
Under the declaration, the countries pledged to achieve the following:
y To facilitate the establishment of partnerships, operators along the
value chain
y To accelerate decarbonization of shipping sector

Environment 43
y To reduce the fuel supply to the shipping sector by establishing
green shipping corridor
y To identify and explore barriers in green corridor establishment.
This includes regulatory framework, information sharing, incentives.
y To consider including green corridors in National Action Plans

National and Global Initiatives

In the News
y Cleaning of rivers is a continuous process and the Central
government supplements the efforts of the State Governments
in addressing the challenges of pollution of rivers by providing
financial and technical assistance.
y Under the National River Conservation Plan (NRCP), project proposals
are received from the State Government from time to time for
taking up pollution abatement works in towns along various rivers,
and are considered for financial assistance on a cost sharing basis
subject to their conformity with the scheme guidelines, pollution
status, prioritizations, appraisal by independent institutions and
46. National river availability of Plan funds.
conservation
plan (NRCP) About National river conservation plan (NRCP)
y The river cleaning programme in the country was initiated with the
launching of the Ganga Action Plan (GAP) in 1985.
y In 1995, the Ganga Action Plan was expanded to cover other rivers
under the National River Conservation Plan (NRCP).
y Objective of NRCP: To improve the water quality of the rivers,
which are the major water sources in the country, through the
implementation of pollution abatement works.
y Works under NRCP include: collection, transportation and
treatment of municipal sewage, River Front Development (RFD),
Low Cost Sanitation (LCS), Electric Crematoria, Improved Wood
Based Crematoria etc.

National Water Mission or Jal


Jeevan Mission
y Jal Jeevan Mission (Rural): Objective is to provide 55 liters of water
per person per day to every rural household through Functional
47. National Water Household Tap Connections (FHTC) by 2024.
Mission ⚪ Har Ghar Jal status, which involves the provision of tap water
supply to all rural households, has been attained by 9 states
and Union Territories.
 These are: Punjab, Haryana, Himachal Pradesh, Goa, Andaman
and Nicobar Islands, Telangana, Gujarat, Puducherry, Daman
and Diu and Dadra Nagar Haveli.

44 Environment
y World Water Week Theme: Organized by Stockholm International
Water Institute. This year’s theme, “Seeds of Change: Innovative
Solutions for a Water-Wise World,” spotlights innovation in
addressing current water challenges.

In the News
y The Union Cabinet approves categorizing the Coalition for Disaster
Resilient Infrastructure (CDRI) as an “International Organization”.
⚪ It also approved the signing of the Headquarters Agreement
further granting it the exemptions, immunities, and privileges
contemplated by the United Nations (Privileges and Immunities)
Act, 1947, which means that member’s property and assets
wherever located and by whomsoever held, shall enjoy immunity
from every form of legal process except in any particular case it
has expressly waived its immunity.

Significance
y Expert Consultations:
⚪ It would help in delegating experts to other countries that are
particularly vulnerable to disaster risk and/or require assistance
with post-disaster recovery, as well as bringing experts from
member countries to India for similar purposes.
⚪ It will make technical expertise available to assist countries
in developing resilient infrastructure in accordance with their
48. Coalition for disaster and climate risks and resources;
Infrastructure ⚪ It will provide assistance to countries in developing appropriate risk
(CDRI) governance arrangements and resilient infrastructure strategies.
y Enhanced Funding and Cooperation:
⚪ It will help in deploying funds globally and receiving contributions
from member countries for CDRI activities.
⚪ It will provide all possible assistance to member countries
in upgrading their systems to ensure disaster and climate
resilience of existing and future infrastructure, in accordance
with the Sustainable Development Goals (SDGs), the Paris
Climate Agreement, and the Sendai Framework for Disaster
Risk Reduction.
⚪ It will use international engagement to foster disaster-resilient
infrastructure at home and providing an opportunity for Indian
scientific and technical institutions as well as infrastructure
developers to interact with global experts

Coalition for Disaster Resilient Infrastructure (CDRI)


y CDRI is a global partnership of national governments, United
Nations agencies and programmes, multilateral development banks
and financing mechanisms, the private sector, and academic and
research institutions.

Environment 45
y It was launched in 2019, at the United Nations Climate Action
Summit in New York.
⚪ It is the Government of India’s second major global initiative
after the International Solar Alliance, and it demonstrates India’s
leadership in climate change and disaster resilience issues.
y Aim: Increase the resilience of infrastructure systems to climate
and disaster risks, thereby ensuring sustainable development.

Members:
y Since its inception, 31 countries, 6 international organisations and
2 private sector organisations have joined CDRI as members.
⚪ 6 International Organisations: Asian Development Bank (ADB),
World Bank Group, United Nations Development Programme
(UNDP), United Nations Office for Disaster Risk Reduction
(UNDRR), European Union, European Investment Bank.
⚪ 2 Private Sector Organisations: The Private Sector Alliance for
Disaster Resilient Societies and Coalition for Climate Resilient
Investment.
y CDRI has steadily increased its membership by attracting a diverse
range of economically advanced, developing, and vulnerable
countries to climate change and disasters.

y The ecological flow, or e-flow notification as it is called, specifies


that the upper stretches of the Ganga — from its origins in the
glaciers and until Haridwar — would have to maintain:
⚪ 20% of the monthly average flow of the preceding 10-days
between November and March,
⚪ 25% of the average in the dry season

Why in News
y According to the Central Water Commission (CWC), 4 of the 11
hydro power projects on the upper reaches of the river Ganga’s
tributaries are violating Ganga ecological flow (e-flow) norms.
49. E-flow norms for
y The non-compliant ones are:
river Ganga
⚪ Vishnuprayag Hydroelectric project - Alaknanda
⚪ Srinagar Hydroelectric project - Alaknanda
⚪ Maneri Bhali Phase 2 - Bhagirathi
⚪ Pashulok - Ganga Mainstream
y Violating the e-flow norms can mean closure of the project or a
hefty fine.

E-flow norms
y The Central Government under the Environment (Protection) Act,
1986 has notified the minimum environmental flows for the River
Ganga that has to be maintained at various locations on the river.

46 Environment
y This is to ensure that the river has at least the minimum required
environmental flow of water even after the river flow gets diverted
by projects and structures for purposes like irrigation, hydropower,
domestic and industrial use etc. This is an important step taken
towards maintaining the uninterrupted or Aviral Dhara of the river.
y Environmental flows are the acceptable flow regimes that are
required to maintain a river in the desired environmental state or
predetermined state.
y The Central Water Commission is the designated authority and will
be responsible for supervision, monitoring, regulation of flows and
reporting of necessary information to the appropriate authority as
and when required and also take emergent decisions about the
water storage norms in case of any emergency.

Innovation Roadmap of the Mission


Integrated Biorefineries
y It has been developed with inputs from Brazil, Canada, European
Commission and the UK.
y Mission Innovation is a global initiative to catalyse action and
investment in research, development and demonstration to make
clean energy affordable, attractive and accessible to all this decade.
50. Innovation y A major futuristic PPP (Public Private Participation) mode initiative
Roadmap of for ‘Clean Energy’ with the full launch of “Mission Integrated
the Mission Biorefineries”.
Integrated
Biorefineries Goal
(IRMIB) y Develop and demonstrate innovative solutions to accelerate the
commercialization of integrated biorefineries, with a target of
replacing 10% of fossil-based fuels, chemicals and materials with
bio-based alternatives by 2030
y It will advance sustainable biorefining pathways and technologies
to support the development and commercialization of bio-based
fuels, chemicals and materials, by also considering process energy
demands.

y ‘Mission LiFE’ (Lifestyle For Environment) is a global initiative for a


sustainable and healthy lifestyle.
y Objective: Help the world in its fight against climate change
and lead to a sustainable way of life to achieve the sustainable
development goals(SDG).
51. Mission LiFE
y It aims to make the fight against climate change democratic with
(Lifestyle for
the contribution of everyone in per own capacity.
Environment)
Mission LIFE
y The idea of life promotes an environmentally conscious lifestyle
that focuses on ‘mindful and deliberate utilization’ instead of
‘mindless and wasteful consumption.

Environment 47
y Key points:
⚪ Promoting Environmentally Conscious Lifestyles: Circular
economy where the concept of ‘Reduce, Reuse and Recycle’
can help in striking a balance between development, economic
growth, and sustainability.
⚪ Trusteeship model: Everyone as trustee of the environment. A
trustee is someone who does not allow indiscriminate use of
resources. A trustee works as a nurturer and not as an exploiter.
⚪ Mobilising Global collective action: The Mission emboldens the
spirit of the P3 model, i.e., Pro Planet People, as it is premised
on the basic principles of ‘Lifestyle of the planet, for the planet
and by the planet’.

y The CBAM regulation is an import tariff to put a fair price on the


carbon emitted during the production of carbon intensive goods
that are entering the EU, and to encourage cleaner industrial
production in non-EU countries
y The transitional reporting phase begins October 1, 2023 to December
31, 2025.

About CBAM
y CBAM is part of the “Fit for 55 in 2030 package”, which is the EU’s
plan to reduce greenhouse gas emissions by at least 55% by 2030
compared to 1990 levels in line with the European Climate Law.
y It is compliant with WTO-rules.

Objectives
y CBAM will ensure its climate objectives are not undermined by
52. Carbon Border
carbon-intensive imports and spur cleaner production in the rest
Adjustment
of the world.
Mechanism
(CBAM) Implementation
y The CBAM will be implemented by requiring importers to declare
the quantity of goods imported into the EU and their embedded
Greenhouse Gas emissions on an annual basis.
y To offset these emissions, importers will need to surrender a
corresponding number of CBAM certificates, the price of which
will be based on the weekly average auction price of EU Emission
Trading System (ETS) allowances in €/tonne of CO2 emitted.

Functioning
y By confirming that a price has been paid for the embedded carbon
emissions generated in the production of certain goods imported
into the EU, the CBAM will ensure the carbon price of imports is
equivalent to the carbon price of domestic production, and that
the EU’s climate objectives are not undermined.

48 Environment
Energy Conservation (Amendment) Act, 2022
y Parliament has passed the Energy Conservation (Amendment) Bill,
2022. The Bill seeks to amend the Energy Conservation Act 2001 to
facilitate the achievement of COP-26 goals and introduce concepts
such as mandated use of non-fossil sources and carbon credit
trading to ensure faster decarbonisation of the Indian economy.
This Bill empowers the central government to specify a carbon
credit trading scheme and provide for Energy Conservation and
Sustainable Building Code.

Key Features
y Carbon credit trading: The Bill empowers the central government
to specify a carbon credit trading scheme.
⚪ Carbon credit implies a tradeable permit to produce a specified
amount of carbon dioxide or other greenhouse emissions.
⚪ The central government or any authorised agency may issue
carbon credit certificates to entities registered and compliant
with the scheme allowing entities to trade the certificates.
y Obligation to use non-fossil sources of energy: The Act empowers
the central government to specify energy consumption standards
with designated consumers required to meet a minimum share of
53. Energy energy consumption from non-fossil sources.
Conservation ⚪ Different consumption thresholds may be specified for different
(Amendment) non-fossil sources and consumer categories.
Act, 2022 ⚪ Designated consumers include: (i) industries such as mining,
steel, cement, textile, chemicals, and petrochemicals, (ii)
transport sector including Railways, and (iii) commercial
buildings, as specified in the schedule.
⚪ Failure to meet this obligation will be punishable with a penalty
of up to Rs 10 lakh. It will also attract an additional penalty
of up to twice the price of oil equivalent of energy consumed
above the prescribed norm.
y Energy conservation code for buildings: The Act empowers
the central government to specify Energy Conservation and
Sustainable Building Code for buildings. The code prescribes
energy consumption standards in terms of area.
⚪ The Code will apply to the office and residential buildings having
a minimum connected load of 100 kilowatt (kW)
y Standards for vehicles and vessels: Energy consumption standards
may be specified for vehicles and ships for equipment and
appliances which consume, generate, transmit, or supply energy.

y Composition of the governing council of BEE: The Act provides for


the setting up of the Bureau of Energy Efficiency (BEE).
⚪ The Bureau has a governing council with between 31-37
members, instead of 20-26 previously. These include:
 Secretaries of 12 departments,

Environment 49
 Representatives of regulatory authorities such as the Central
Electricity Authority, and the Bureau of Indian Standards,
 Up to 7 members representing industries and consumers.

Objective
y To motivate farmers to adopt chemical free farming and enhance
the reach of natural farming by up-scaling the Bhartiya Prakritik
Krishi Paddati (BPKP).
y The plan requires:
⚪ Behavioral change in farmers to shift from chemical based
inputs to cow based locally produced inputs and
⚪ Requires continuous creation of awareness, training, handholding
and capacity building of farmers in the initial years.

Natural Farming
y Natural Farming is a way of chemical free farming based on desi
cow and locally available resources, with no chemical fertilizers
and pesticides and promotes traditional indigenous practices
which give freedom to farmers from externally purchased inputs.
⚪ It is largely based on on-farm biomass recycling with major
stress on biomass mulching, use of on-farm desi cow dung-
54. National Mission urine formulation; managing pests through diversity, on-farm
on Natural botanical concoctions and exclusion of all synthetic chemical
Farming inputs directly or indirectly and emphasis is given on improving
natural nutrient cycling and increase in organic matter in the
soil.

Organic Farming
y A system of farm design and management to create an ecosystem
which can achieve sustainable productivity without use of artificial
off farm input such as chemical fertilizer and pesticides.
⚪ It is considered to be climate friendly farming practices
promoting low external input usage, recycling, reuse and
reduced use of synthetics in farming.

Climate Smart agriculture


y It is a broader concept which includes all environment friendly
agricultural approaches like integrated farming systems,
conservation agriculture, natural farming, organic farming, precision
agriculture, regenerative agriculture, reclamation of degraded soils
and reduced food loss and waste to achieve sustainable agriculture.

50 Environment
y e-AMRIT (Accelerated e-Mobility Revolution for India’s
Transportation) portal for creating awareness about electric
mobility in India
y It was launched by India at the COP26 Summit
⚪ It is the result of a joint initiative between NITI Aayog and the
UK Government.

Objective
y To serve as a ‘one-stop site’ to provide all the information related
to the adoption of electric vehicles in India

Purpose
55. E-amrit portal y It is a web portal functioning as a one-stop destination helping
users to:
⚪ Switch to electric vehicles by providing insights about electric
vehicle technologies, types of electric vehicles, insurance
options, and financing options.
⚪ Set up the electric vehicle or associated enterprise by providing
insights on the key initiatives of central and state governments.
⚪ Assess the benefits of electric vehicles with uniquely designed
tools to determine the users’ savings with electric vehicles
compared to petrol/diesel vehicle
⚪ Get all the information on the Indian electric vehicle market
and industry and the key developments driving the e-mobility
ecosystem forward.

y It is one of the eight national missions under the National Action


Plan on Climate Change (NAPCC).
y The Mission has been implemented since 2011.

Aim:
y To strengthen the market for energy efficiency by creating a
conducive regulatory and policy regime
y Envisage fostering innovative and sustainable business models in
56. National mission
the energy efficiency sector.
for enhanced
energy efficiency Schemes under NMEEE:
(NMEEE)
y Perform Achieve and Trade Scheme (PAT)
y Market Transformation for Energy Efficiency (MTEE)
y Energy Efficiency Financing Platform (EEFP)

Components of NMEEE
National mission for enhanced energy efficiency (NMEEE) consists
of four initiatives to enhance energy efficiency in energy-intensive
industries, which are as follows:

Environment 51
y Perform Achieve and Trade Scheme (PAT): Implementing a market-
assisted compliance mechanism to accelerate the implementation
of cost-effective improvements in energy efficiency in large energy-
intensive industries.
y Market Transformation for Energy Efficiency (MTEE): Accelerating
the shift to energy-efficient appliances in specific applications
through innovative measures to make the products more affordable.
y Energy Efficiency Financing Platform (EEFP): Facilitating Financial
Institutions to invest in Energy Efficiency Projects and Programmes.
y Framework for Energy Efficient Economic Development (FEEED):
Developing fiscal instruments to leverage financing for Energy
Efficiency through risk mitigation.
y Framework for Energy Efficient Economic Development (FEEED)

Implementation agencies:
y BEE: Bureau of Energy Efficiency.
y EESL: Energy Efficiency Services Limited

y It is one of the three components of the Integrated Development


of Wildlife Habitats (IDWH).

Objective
y Provide support to protected areas (national parks, wildlife
sanctuaries, conservation reserves and community reserves except
57. Species recovery tiger reserves
programme y protection of wildlife outside protected areas and recovery
programmes for saving critically endangered species and habitats.
Functioning
y MoEFCC identified 16 terrestrial and 6 aquatic species with the
objective of saving critically endangered species/ecosystems to
ensure their protection outside Protected Areas, across the wider
landscape/seascape.

y The Antarctic Treaty was signed between 12 countries in Washington


on 1st December 1959 for making the Antarctic Continent a
demilitarized zone to be preserved for scientific research only.
⚪ Antarctica is the land and ice shelves south of 60°S latitude.
y Headquartered in Buenos Aires, Argentina.
58. Antarctic treaty y Original signatories: Argentina, Australia, Belgium, Chile, France,
Japan, New Zealand, Norway, South Africa, the Union of Soviet
Socialist Republics, the UK and the US.
⚪ India became a member of this treaty in 1983.
y The Antarctic treaty remains the only example of a single treaty
that governs a whole continent.

52 Environment
y It is a guidebook developed by the United Nations to provide
standards for incorporating natural capital and environmental
quality into national accounting systems.
y Other accounting standards globally:
⚪ Happy Planet Index
⚪ Gross National Happiness Index of Bhutan
y It is an integrated and comprehensive statistical framework for
59. Seea- ecosystem organizing data about habitats and landscapes, measuring the
accounting ecosystem services, tracking changes in ecosystem assets, and
linking this information to economic and other human activity.
Ecosystem Accounting
y It works by compiling data from the below mentioned accounts:
⚪ Ecosystem extent
⚪ Ecosystem condition
⚪ Ecosystem services
⚪ Monetary ecosystem asset

y Conservation Assured Tiger Standards (CATS) accreditation was


announced for 14 out of India’s 51 tiger reserves by the MoEFCC.
Conservation assured tiger standards(CA|TS)
y CATS are a globally accepted conservation tool that sets best
60. Conservation practices and standards to manage tigers and assessments to
assured tiger benchmark progress.
standards(CA|TS) y Coverage: Tigers are the first species selected for the initiative.
⚪ It is being adopted for use beyond tigers, including potentially
jaguars, lions and freshwater dolphins.
y Implementation: Global Tiger Forum and WWF India are
implementing partners of the National Tiger Conservation
Authority (NTCA) for CATS assessment in India.

y The Aravali Green wall project was launched by the Ministry of


Environment, Forest and Climate Change. It has been inspired by
Africa’s Great Green Wall (GGW) programme.
y Green Wall Project is a major initiative to create green corridors
in 5 km buffer area around Aravalli Hill
y Range in four states - Haryana, Rajasthan, Gujarat and Delhi.
⚪ The project involves planting native species of trees and shrubs
61. Aravali Green on scrubland, wasteland and degraded forest land
wall project ⚪ The program also aims at rejuvenating and restoring surface
water- bodies such as ponds, lakes and streams.
Great Green Wall (GGW) programme:
y It is an initiative launched in 2007 in the Sahel region bordering
Africa’s Sahara Desert, to combat desertification and land
degradation.
y The GGW project aims to restore 100 million hectares of degraded
land by 2030.

Environment 53
y Maharashtra State Board for Wildlife (SBWL) approves to declare e
2,011 square kilometre Angria Bank as a protected area under the
Maritime Zones Act, 1976.
About Angria Bank
y Angria Bank is a submerged plateau situated 105 km offshore from
62. Angria Bank the southern coast of Maharashtra with depth of 20m to 400m.
y It supports a large extent of coral reefs and algal habitats, spanning
across 650 square kilometres that harbours a high diversity of
associated flora and fauna.
y The reefs also host diverse functional groups that are necessary
for a stable ecosystem representing a resilient reef.

News
y The International Big Cats Alliance (IBCA) was launched to mark
50 years of Project Tiger with an investment of $100 million.
y The proposed alliance will provide assured support over five years
63. International Big with guaranteed funding of over Rs 800 crore.
Cat Legislation y The group will work towards the protection of the seven big cats —
tiger, lion, leopard, snow leopard, puma, jaguar and cheetah.
⚪ Membership to the alliance will be open to 97 “range” countries,
which contain the natural habitat of these big cats, as well as
other interested nations, international organizations, etc.

In July 2023, the Commission for Air Quality Management in NCR &
Adjoining Areas (CAQM) announced revision of the Graded Response
Action Plan (GRAP) which came into force w.e.f. 1st October, 2023 in
the entire National Capital Region.

About GRAP
y GRAP is a set of emergency measures that kick in to prevent further
deterioration of air quality once it reaches a certain threshold in
the Delhi-NCR region.
y It was approved by the Supreme Court in 2016 after the Supreme
Court’s order in the matter of M. C. Mehta vs. Union of India (2016)
64. Graded Response
and notified in 2017.
Action Plan
Implementation
y From 2021 onwards, the GRAP is being implemented by the CAQM.
y CAQM relies on air quality and meteorological forecasts by the
Indian Institute of Tropical Meteorology (IITM) and the India
Meteorological Department (IMD).

Commission for Air Quality Management


y Formed by an ordinance, “Commission for Air Quality Management
(CAQM) in National Capital Region and Adjoining Areas Ordinance
2020”, in October 2020.

54 Environment
y The Commission will be a statutory authority. It will supersede
bodies such as the central and state pollution control boards of
Delhi, Punjab, Haryana, UP and Rajasthan.
y Composition: The new 18-member Commission brings together
the Centre, states, and other stakeholders on one collaborative
platform.
y The Commission will have the power to impose a fine of up to Rs 1
crore and imprisonment of up to 5 years in case its directions are
contravened.

y The Reserve Bank of India (RBI) said that the maiden Sovereign
Green Bonds (SGrBs) would be issued in two tranches for an
aggregate amount of Rs 16,000 crore.
y The proceeds will be utilised for funding public sector projects
seeking to reduce carbon emissions.
Background:
y Prime Minister Narendra Modi at COP26 in November 2021 had
announced India’s commitments under ‘Panchamrit’ to reduce
carbon emissions.
⚪ Panchamrit is a set of five commitments that the Prime Minister
made at the Conference of Parties (COP26) held at Glasgow, UK.
⚪ These five commitments are –
 To raise the non-fossil fuel based energy capacity of the
country to 500 GW by 2030.
 By 2030, 50% of the country’s energy requirements would
be met using renewable energy sources.
 The country will reduce the total projected carbon emission
65. India’s First
by one billion tonnes between now and the year 2030.
Sovereign Green
 The carbon intensity of the economy would be reduced to
Bond Framework
less than 45% by 2030.
 India would become carbon neutral and achieve net zero
emissions by the year 2070.
What are Green Bonds?
y Green bonds are issued by companies, countries and multilateral
organisations to exclusively fund projects that have positive
environmental or climate benefits and provide investors with fixed
income payments.
y The projects can include renewable energy, clean transportation
and green buildings, among others.

Example of Green Bonds:


y The World Bank is a major issuer of green bonds and issued $14.4
billion of green bonds between 2008 and 2020.
y These funds have been used to support 111 projects around the
world, largely in renewable energy and efficiency (33%), clean
transportation (27%), and agriculture and land use (15%).

Environment 55
y By the end of 2020, 24 national governments had issued Sovereign
Green, Social and Sustainability bonds totalling a cumulative $111
billion.

India’s Sovereign Green Bonds Framework


y Under the framework, the Finance Ministry will, every year, inform
the RBI about spending on green projects for which the funds
raised through these bonds will be used.

Features of the SGrBs:


y Issuance Method –
⚪ SGrBs will be issued through Uniform Price Auction (a public
sale in which a fixed number of similar things are sold at the
same price).
y Eligibility for Repurchase Transactions (Repo) –
⚪ SGrBs will be eligible for Repurchase Transactions (Repo).
⚪ SGrBs will also be reckoned as eligible investment for Statutory
Liquidity Ratio (SLR) purpose.
y Tradability –
⚪ SGrBs will be eligible for trading in the secondary market.

y Investment by Non-residents –
⚪ SGrBs will be designated as specified securities under the ‘Fully
Accessible Route’ for investment in Government Securities by
non-residents.

Eligible Projects
y All eligible green expenditures will include public expenditure
undertaken by the government in the form of investment, subsidies,
grants-in-aid, or tax foregone (or a combination of all or some of
these) or select operational expenditures.
y R&D expenditures in public sector projects that help in reducing
the carbon intensity of the economy and enable the country to
meet its Sustainable Development Goals (SDGs) are also included
in the framework.
y The eligible expenditures will be limited to government
expenditures that occurred maximum 12 months prior to issuance
of the green bonds.
y Sectors not included –
⚪ Nuclear power generation, landfill projects, alcohol/weapons/
tobacco/gaming/palm oil industries and hydropower plants
larger than 25 MW have been excluded from the framework.

Where will the proceeds go?


y The proceeds from the green bonds issuance will be deposited
in the Consolidated Fund of India (CFI) in line with the regular
treasury policy, and then funds from the CFI will be made available
for the eligible green projects.

56 Environment
Implementing Agency:
y The Ministry of Finance has constituted a Green Finance Working
Committee (GFWC) including members from relevant ministries
and chaired by the Chief Economic Advisor.
y The GFWC will meet at least twice a year to support the Ministry
of Finance with selection and evaluation of projects and other
work related to the Framework.

Recently, the UN-Habitat has identified issues associated with Jaipur


city like multi hazard vulnerabilities, weak mobility and Green-Blue
economy and has laid out a plan to increase sustainability in the city.
y The urban problems which persist in Jaipur are the same for other
cities.
y UN-Habitat has based its findings on sustainable cities integrated
approach pilot project and a “sustainable urban planning and
management” component was implemented in partnership with
the Jaipur Development Authority and Jaipur Greater Municipal
Corporation.

⚪ The project has received funding from the Global Environment


Facility (GEF-6) to estimate the carbon sequestration potential
of Indian cities.

Findings of the Project?


y Jaipur got an overall sustainability rating of three on the Urban
66. Sustainable
Sustainability Assessment Framework (USAF) based on the
Cities Integrated
information collected for 87 of its 131 parameters.
Approach Pilot
y The UN-Habitat highlighted the following problems which are faced
(SCIAP) Project
by the city:
⚪ Weak access to a public transportation system, with less
number of buses and poor route delineation.
⚪ Extreme levels of drought during summer and also urban
floods.
⚪ Lack green cover which has resulted in the urban heat island
effect that has disrupted biodiversity.

What is the UN-Habitat?


y The United Nations Human Settlements Programme is the United
Nations programme for human settlements and sustainable urban
development.
y It was established in 1978 as an outcome of the First UN Conference
on Human Settlements and Sustainable Urban Development
(Habitat I) held in Vancouver, Canada, in 1976.
y UN-Habitat maintains its headquarters at the United Nations
Office at Nairobi, Kenya.

Environment 57
y It is mandated by the United Nations General Assembly to promote
socially and environmentally sustainable towns and cities with the
goal of providing adequate shelter for all.
y It is a member of the United Nations Development Group. The
mandate of UN-Habitat derives from the Habitat Agenda, adopted
by the United Nations Conference on Human Settlements (Habitat
II) in Istanbul, Turkey, in 1996.
y The twin goals of the Habitat Agenda are:
⚪ Adequate shelter for all
⚪ The development of sustainable human settlements in an
urbanizing world

What is the Global Environmental Facility(GEF)?


y It is an independently operating financial organization.
y GEF is a multilateral financial mechanism that provides grants to
developing countries for projects that benefit the global environment
and promote sustainable livelihoods in local communities.
y It was set up as a fund under World Bank in 1991.
y In 1992, at the Rio Earth Summit, the GEF was restructured and
moved out of the World Bank system to become a permanent,
separate institution.
y Since 1994, however, the World Bank has served as the Trustee of
the GEF Trust Fund and provided administrative services.
y It is based in Washington DC, United States.

Recently, the National Institute of Ocean Technology, an autonomous


institute under the Union Ministry of Earth Sciences (MoES) is
establishing an Ocean Thermal Energy Conversion Plant with a capacity
of 65 kilowatts (kW) in Kavaratti, Lakshadweep.
y The plant is the first of its kind in the world as it will generate
drinking water from sea water using indigenous technology, green
energy and environmentally friendly processes.

Ocean Thermal Energy Conversion:


y Ocean Thermal Energy Conversion (OTEC) is a process for producing
67. Ocean Thermal
energy by harnessing the temperature differences (thermal
Energy
gradients) between ocean surface waters and deep ocean waters.
⚪ Oceans are huge heat reservoirs as they cover almost 70% of
Earth’s surface.
y Researchers focus on two types of OTEC technologies-
⚪ Closed cycle method - where a working fluid (ammonia) is
pumped through a heat exchanger for evaporation and the
steam runs a turbine.
 The vapour is turned back to fluid (condensation) by the cold
water found at the depths of the ocean where it returns to
the heat exchanger.

58 Environment
⚪ Open cycle method - where the warm surface water is
pressurized in a vacuum chamber and converted to steam
which runs the turbine. The steam is then condensed using
cold ocean water from lower depths.
y Historical perspective:
⚪ India initially had planned to set up an OTEC plant way back
in 1980, off the Tamil Nadu coast. However, with the foreign
vendor closing down its operation, it had to be abandoned.

How does an OTEC Plant Work?:


y As the energy from the sun heats the surface water of the ocean.
In tropical regions, surface water can be much warmer than deep
water.
y This temperature difference can be used to produce electricity
and desalinate ocean water.
⚪ Ocean Thermal Energy Conversion (OTEC) systems use a
temperature difference (of at least 77°F) to power a turbine to
produce electricity.
⚪ Warm surface water is pumped through an evaporator containing
a working fluid. The vaporized fluid drives a turbine/generator.
⚪ Then the vaporized fluid is turned back to a liquid in a condenser
cooled with cold ocean water pumped from deeper into the
ocean.
⚪ OTEC systems use seawater as the working fluid and can use
condensed water to produce desalinated water.

Significance:
y Two of the biggest advantages of OTEC are that it produces clean
environmentally friendly renewable energy and, unlike solar plants
which can’t work at night and wind turbines which only work when
it’s windy, OTEC can produce energy at all times.

Recent Initiatives of the Government


y Deep Sea Mining: The MoES is developing technologies for mining
deep sea resources like polymetallic nodules from the Central
Indian Ocean at a water depth of 5,500 meters.
y Weather Forecasting:
⚪ The ministry is also working on introducing ocean climate
change advisory services for climate risk assessment due to
sea level rise; cyclone intensity and frequency; storm surges
and wind waves; biogeochemistry, and changing harmful algal
blooms in the coastal waters of India.
y Deep Ocean Mission:
⚪ MoES is trying to design and develop a prototype crewed
submersible rated for 6,000 meters of water depth under the
Deep Ocean Mission.

Environment 59
⚪ It will include technologies for underwater vehicles and
underwater robotics.
y DNA Bank:
⚪ There efforts are being made to improve the detection,
sampling and DNA storage of benthic fauna of the northern
Indian Ocean through systematic sampling using a remotely
operated vehicle.

y Sikkim’s recent dam collapse exemplifies blind spots in Dam Safety


Act’s legislation and implementation.
y The Dam Safety Bill introduced in Lok Sabha in July 2019 was
passed by Rajya Sabha in December 2021 to become Dam Safety
Act, 2021.

Major threats to dams in India


y Ageing: With over 6,000 dams in India, ranked third globally in terms
of large dams and approximately 80% of these dams exceeding 25
years of age while 234 surpassing the century mark, ensuring their
safety is of paramount importance.
⚪ For India, 2025 is set to be a big year as more than 1,000 dams
would turn roughly 50 years or older.
y Accumulation of sediments & siltation: Accumulation of sediments
decreases a reservoir’s capacity over the years and determines a
reservoir’s life expectancy.
⚪ Siltation, which is the accumulation of silt and debris behind
the reservoir, also leads to a reduction in the storage capacity
of the dams.
68. Dam safety Bill ,
y Structural issues: India’s dams are more vulnerable to deterioration
2019
because a large proportion of them are earthen–built by compacting
successive layers of earth, and not concrete–and are hence more
prone to ageing.
y Flooding: The country gets concentrated rainfall every year for a
designated time period as opposed to distributed rainfall, which
contributes to the dams’ vulnerability.
⚪ In India, the downstream areas are often exposed to flood
disasters & flooding has caused 44% of dam failures in India.
y Seismic threat: Some of the Himalayan dam systems, including the
Tehri Dam, are in an active seismic area given that the Himalayan
mountain system is constantly changing and growing giving rise to
several tectonic movements.
y Dam Safety Act, 2021: A Regulatory Framework:
⚪ Enactment of the Dam Safety Act, 2021, by the Union
Government.
⚪ Focuses on proper surveillance, inspection, operation, and
maintenance of specified dams.
⚪ Aims to prevent dam failure-related disasters and establish an
institutional mechanism for safe functioning.

60 Environment
⚪ Institutional Mechanism:
 National Committee on Dam Safety (NCDS):
• Formation of the National Committee on Dam Safety at
the national level.
• Responsible for evolving dam safety policies and
recommending essential regulations.
• Provides a strategic platform for ensuring uniform safety
standards.
 National Dam Safety Authority (NDSA):
• Creation of the National Dam Safety Authority as a
regulatory body.
• Tasks include implementing policies of the National
Committee on Dam Safety.
• Offers technical assistance to State Dam Safety
Organisations (SDSO) and resolves inter-state disputes.

 State-level Dam Safety Measures:


• Empowerment of State Governments to establish State
Committee on Dam Safety.
• Creation of State Dam Safety Organisations responsible
for enforcing dam safety standards.
• Renders crucial instructions to dam owners regarding
safety protocols and remedial actions.

y National Hydrology Project (NHP):


⚪ National Hydrology Project (NHP) s designed with four major
components: Water Resources Monitoring System, Water
Resources Information System, Water Resources Operations
and Planning System, and Institutional Capacity Enhancement.
⚪ The project aims to enhance water resource management
capabilities across the country.
⚪ Supports studies related to flood forecasting undertaken by
Implementing agencies.

The Ministry of New & Renewable Energy has allowed households to get
rooftop solar panels installed by themselves or by any vendor of their
choice and a photograph of the installed system for distribution utility is
sufficient to avail benefits or subsidy under the Rooftop solar scheme.
Implemented by the Ministry of New and Renewable Energy.
Presently under implementation is the Grid-Connected Rooftop Solar
69. Rooftop Solar
Scheme (Phase II): It aims to achieve a cumulative capacity of 40,000
Programme
MW from Rooftop Solar Projects by the year 2022.
This scheme is being implemented in the state by distribution
companies (DISCOMs).
y Under this scheme the Ministry is providing a 40% subsidy for the
first 3 kW and 20% subsidy beyond 3 kW and upto 10 kW of solar
panel capacity.

Environment 61
y The residential consumer has to pay the cost of rooftop solar
plants by reducing the subsidy amount given by the Ministry as per
the prescribed rate to the vendor.

Major objective of the programme


y To promote the grid-connected SPV rooftop and small SPV power
generating plants among the residential, community, institutional,
industrial and commercial establishments.
y To mitigate the dependence on fossil fuel based electricity
generation and encourage environment-friendly Solar electricity
generation.
y To create an enabling environment for investment in the solar
energy sector by the private sector, state government and the
individuals.
y To create an enabling environment for the supply of solar power
from rooftop and small plants to the grid.

Schemes for Promoting Solar Energy


y Rooftop Solar Scheme: To generate solar power by installing
solar panels on the roof of the houses, the Ministry of New and
Renewable Energy is implementing Grid-connected Rooftop Solar
Scheme (Phase II).
⚪ It aims to achieve a cumulative capacity of 40,000 MW from
Rooftop Solar Projects by 2022.
y Kisan Urja Suraksha evam Utthaan Mahabhiyan: The scheme
covers grid-connected Renewable Energy power plants (0.5 – 2
MW)/Solar water pumps/grid connected agriculture pumps.
y International Solar Alliance (ISA): The ISA, is an Indian initiative
that was launched by the Prime Minister of India and the President
of France on 30th November 2015 in Paris, France on the side-lines
of the Conference of the Parties (COP-21), with 121 solar resource
rich countries lying fully or partially between the tropic of Cancer
and tropic of Capricorn as prospective members.
y One Sun, One World, One Grid: It has been taken up under the
technical assistance program of the World Bank. Its objective is to
aid in developing a worldwide grid through which clean energy can
be transmitted anywhere, anytime.
y National Solar Mission (A part of National Action Plan on Climate
Change).

The Union Government has approved a Rs 19,744 crore National Green


Hydrogen mission that aims to make India a ‘global hub’ for using,
70. National
producing and exporting green hydrogen.
Hydrogen
Mission-
In Apr 2022: India’s first 99.999% pure green hydrogen plant was
Hydrogen Energy
commissioned by Oil India in Jorhat, Assam. In Jan 2023: Cabinet
approved National Green Hydrogen Mission.

62 Environment
What is Green Hydrogen?
y Green hydrogen(GH2) is Hydrogen(H₂) produced by splitting
water(H₂O) into hydrogen and oxygen(O₂) using renewable electricity.
y In comparison, Grey hydrogen is traditionally produced from
methane (CH4) releasing CO₂, a GHG.
y Blue hydrogen is similar to Grey Hydrogen with the additional
technologies necessary to capture the CO₂.
National Green Hydrogen Mission
y It is a program to incentivise the commercial production of green
hydrogen and make India a net exporter of the fuel.
y The Mission will facilitate demand creation, production, utilization
and export of Green Hydrogen.
y Sub Schemes:
⚪ Strategic Interventions for Green Hydrogen Transition
Programme (SIGHT):
 It will fund the domestic manufacturing of electrolysers
and produce green hydrogen.
⚪ Green Hydrogen Hubs:
 States and regions capable of supporting large scale
production and/or utilization of hydrogen will be identified
and developed as Green Hydrogen Hubs.

Objective:
y Developing green hydrogen production capacity of at least 5 MMT
(Million Metric Tonne) per annum, alongside adding renewable
energy capacity of about 125 GW (gigawatt) in India by 2030.
y It aims to entail over Rs 8 lakh crore of total investments and is
expected to generate six lakh jobs.
y It will also lead to a cumulative reduction in fossil fuel imports by
over Rs 1 lakh crore and an abatement of nearly 50 MT of annual
greenhouse gas emissions.

Nodal Ministry:
y Ministry of New and Renewable Energy

Potential:
y India has a favorable geographic location and abundance of
sunlight and wind for the production of green hydrogen.
y Green hydrogen technologies are being promoted in sectors where
direct electrification isn’t feasible.
y Heavy duty, long-range transport, some industrial sectors and
long-term storage in the power sector are some of these sectors.
y The nascent stage of this industry allows for the creation of regional
hubs that export high-value green products and engineering,
procurement and construction services.

Environment 63
State-run explorer Oil and Natural Gas Corporation (ONGC) will be
participating to generate electricity through Geothermal Energy at
Puga, a remote valley located in Ladakh, off the road to Chumar on the
de-facto border with China.

Geothermal Energy
y Geothermal energy is the heat from the earth. This heat is used for
bathing, to heat buildings, and to generate electricity.
y The word geothermal comes from the Greek words geo (earth)
and therme (heat), and geothermal energy is a renewable energy
source because heat is continuously produced inside the earth.
y Sources:
⚪ Hot water or steam reservoirs deep in the earth are accessed
by drilling.
⚪ Geothermal reservoirs located near the earth’s surface, mostly
located in the western U.S., Alaska, and Hawaii.
⚪ The shallow ground near the Earth’s surface that maintains a
relatively constant temperature of 50-60°F.
y Uses:
⚪ Hot water and steam from reservoirs can be used to drive
generators and produce electricity for consumers.
71. India’s First
⚪ Other applications apply the heat produced from geothermal
Geothermal
directly to various uses in buildings, roads, agriculture, and
Energy Project.
industrial plants.
⚪ The heat can also be used directly from the ground to provide
heating and cooling in homes and other buildings.

Benefits of Geothermal Energy:


y Renewable Source:
⚪ Through proper reservoir management, the rate of energy
extraction can be balanced with a reservoir’s natural heat
recharge rate.
y Continuous Supply:
⚪ Geothermal power plants produce electricity consistently,
running 24 hours per day/7 days per week, regardless of
weather conditions.
y Reduced Import Dependency:
⚪ Geothermal resources can be harnessed for power production
without importing fuel.

y Small Footprint:
⚪ Geothermal power plants are compact and use less land per
GWh (404 m2) than coal (3642 m2) wind (1335 m2) or solar PV
with center station (3237 m2). *

64 Environment
y Clean Energy:
⚪ Modern closed-loop geothermal power plants emit no
greenhouse gasses; life cycle GHG emissions (50 g CO2 eq/
kWhe) are four times less than solar PV, and six to 20 times
lower than natural gas.
⚪ Geothermal power plants consume less water on average
over the lifetime energy output than the most conventional
generation technologies.
y About Puga Valley:
⚪ Puga Valley is situated in the Changthang Valley in the south-
eastern part of Ladakh, about 22 km away from the Salt Lake
Valley.
⚪ It is a region of great significance known for its natural beauty
and geothermal activities.
⚪ Puga is also visited for its hot sulphur spring.
y About Geothermal Project:
⚪ It will be India’s first geothermal energy project and also the
world’s highest at 14,000ft.

⚪ ONGC has started drilling its first well for the project and
encountered high-pressure steam at 100 degrees Celsius with
a discharge rate of 100 tonne geothermal energy per hour,
considered as a good sign for the project.

Benefits:
y It will boost Ladakh’s potential to emerge as one of the country’s
clean energy bowl by expanding the area’s horizon beyond solar or
wind power.
y The pilot plant provides power and heating needs of the nearby
settlements of Tibetan pastoralist refugee settlements at Sumdo
and nearby areas.
y A bigger plant will provide 24X7 supply for the far-flung settlements
and the large defence establishment in the eastern sector, reducing
their dependence on diesel for running generators.
y The plant can also play a vital role as a stabilizer for the 15-gigawatt
solar/wind project being planned in the nearby Morey plains in the
southwest.

Ministry of Fisheries, Animal Husbandry & Dairying laid the Foundation


Stone for a Multi-Purpose Seaweed Park in Tamil Nadu (Valamavur,
Ramanathapuram district).
Clusters of brown Sargassum seaweed reported to be infested by
72. Seaweeds
flesh-eating bacteria were recently found awash in Florida.
y “Seaweed” is the common name for countless species of marine
plants and algae that grow in the ocean as well as in rivers, lakes,
and other water bodies.

Environment 65
y They range in colours from red, green, brown and black and also
vary in size, from microscopic to large underwater forests.
y Seaweeds are generally anchored to the sea bottom or other solid
structures by rootlike “holdfasts,” which perform the sole function
of attachment and do not extract nutrients as do the roots of
higher plants.
y Many show a well-established zonation along the margins of the
seas, where the depth of the water is 50 metres (about 165 feet)
or less.

Application:
y A number of seaweed species are edible, and many are also of
commercial importance to humans.
y Some are used as fertilizers or as sources of polysaccharides.
y The high amount of antioxidants present in seaweed protects the
body from damages caused by free radicals and protect cells from
their impact.
y Animal studies have found that substances found in seaweed
help increase the production of a protein that metabolises fat
effectively.
y Research suggests that consuming seaweed can help in managing
diabetes by balancing the sugar levels in the blood.
y Many seaweeds contain anti-inflammatory and anti-microbial
agents.
y They are effective binding agents (emulsifiers) in such commercial
goods as toothpaste and fruit jelly, and popular softeners
(emollients) in organic cosmetics and skin-care products.

Seaweed Park:
The park aims to promote seaweed cultivation for employment,
value-added products, and conservation. It involves 136 coastal
fishing villages and offers support to entrepreneurs. The Multipurpose
Seaweed Park is a significant investment to promote seaweed
cultivation and research, benefiting scientists, researchers, and local
communities.

India submitted its Long-Term Low Emission Development Strategy


to the United Nations Framework Convention on Climate Change
73. Long-Term (UNFCCC) at ongoing 27th Conference of Parties (COP27) in Sharm el-
Low Emission Sheikh, Egypt.
Development
Strategy About LT-LEDS
The LT-LEDS are qualitative in nature and are a requirement emanating
from the 2015 Paris Agreement.

66 Environment
y As per this, countries must explain how they will transition their
economies beyond achieving near-term Nationally Determined
Contributions (NDCs) targets and work towards the larger climate
objective of cutting emissions by 45% by 2030 and achieve net zero
around 2050.
y The COP26, held in Glasgow in November 2021, required the parties who
have not yet communicated their LT-LEDS to submit them by COP27.
y India’s LT-LEDS was prepared after extensive consultations
with various government entities, state governments, research
institutes and civil society organizations.

The 7 key pillars of the strategy are:


y Low-carbon electricity systems consistent with development
y Integrated, efficient and inclusive low-carbon transport systems,
y Energy and material-efficiency in buildings,
y Decoupling growth from emissions and developing an efficient,
low-emission industrial system,
y CO2 removal and related engineering solutions,
y Enhancing forest cover consistent with socioeconomic and
ecological considerations
y Increasing climate resilience in poverty eradication and employment
creation.

Salient features
y Forest Cover: India has a strong record of enhancing forest and tree
cover in the last three decades alongside high economic growth.
⚪ India’s forest fire incidence is well below global levels, while
its forest and tree cover are a net sink absorbing 15% of CO2
emissions in 2016.
⚪ India is on track to fulfilling its NDC commitment of 2.5 to 3
billion tonnes of additional carbon sequestration in forest and
tree cover by 2030.
y Transition from fossil fuels: The transition from fossil fuels will be
undertaken in a just, smooth, sustainable and all-inclusive manner.
y Transportation sector: Increased use of biofuels, especially
ethanol blending in petrol, the drive to increase electric vehicle
penetration and the increased use of green hydrogen
y Sustainable Urbanization: Future sustainable and climate-resilient
urbanization: smart city initiatives, integrated planning of cities,
effective green building codes.

y Industrial Sector: India’s industrial sector will continue on a strong


growth path in the perspective of ‘Atma Nirbhar Bharat’ and ‘Make
in India’.

Environment 67
⚪ The focus will be on improving energy efficiency by initiatives
such as Perform, Achieve and Trade (PAT) scheme, high levels
of electrification, enhancing material efficiency and recycling
leading to the expansion of the circular economy.
y Transition to low-carbon development: several costs pertaining
to the development of new technologies, new infrastructure, and
other transaction costs.
⚪ The provision of climate finance by developed countries.

United Nations Decade of Ecosystem Restoration in 2021, an even


wider scale of discussions of Nature-based solutions (NbS) for climate
change adaptation strategy at COP26 is envisaged.
Nature-Based Solutions:
y It refers to the sustainable management and use of nature for
tackling socio-environmental challenges.
y The International Union for Conservation of Nature (IUCN) defines
NbS as actions to protect, sustainably manage and restore
natural and modified ecosystems that address societal challenges
effectively and adaptively, while simultaneously providing human
well-being and biodiversity benefits.
y It is further associated with other sector-specific terms like
74. Global Standards
green infrastructure, natural infrastructure, ecological engineering,
on nature based
ecosystem-based mitigation, ecosystem-based adaptation, and
solutions
ecosystem-based disaster risk reduction.
Global Standard for Nbs:
The eight Criteria that make up the IUCN Global Standard for NbS are
all interconnected.
1. Societal changes
2. Design at Scale
3. Biodiversity net gain
4. Economic Feasibility
5. Inclusive Governance
6. Balance Trade offs
7. Adaptive Management
8. Mainstreaming and sustainability

The Environment Ministry recently held a two-day National Stakeholder


Consultation Meeting on Biodiversity Finance Initiative to conserve
India’s biodiversity.

About:
75. BIOFIN y The Biodiversity Finance Initiative (BIOFIN) is a global partnership
that helps the government’s cost, plan and pay for action on
biodiversity conservation and its sustainable use. It was launched
in 2012 by the United Nations Development Programme (UNDP).
y BIOFIN aims to implement comprehensive national resource
mobilization strategies at national level.

68 Environment
Significance:
y BIOFIN makes recommendations to help the countries priority
identify financing mechanisms – regarding aspects such as
institutional requirements, laws and regulations, taxes and fees,
identification of legal thresholds, removal of biodiversity-harmful
incentives, further feasibility studies and implementation plans,
certification processes, public-private-partnerships, voluntary
agreements, etc.

India has been recognized as the pioneer lead country for the
International Maritime Organization (IMO) Green Voyage2050 Project,
a significant stride toward mitigating GreenHouse Gas (GHG) emissions
from ships.

About the Project:


The GreenVoyage2050 Project is a partnership project between the
Government of Norway and IMO launched in 2019 aiming to transform
76. Green Voyage
the shipping industry towards a lower carbon future.
2050 Project
y The Initial IMO Strategy sets out a clear vision to cut total annual
GHG emissions by a minimum of 50% by 2050, relative to 2008
levels.
y The GreenVoyage2050 Project is working with 12 countries:
Azerbaijan, Belize, China, Cook Islands, Ecuador, Georgia, India,
Kenya, Malaysia, Solomon Islands, South Africa, and Sri Lanka.
⚪ The participating countries can be broadly categorised into
“New Pilot Countries” and “Pioneer Pilot Countries”

Environment 69
Ranks & Reports

“Global Methane Assessment: Benefits and costs of Mitigating


Methane Emissions” was published by the United Nations Environment
Programme (UNEP) in association with the Climate and Clean Air
Coalition.

Key Findings
y There has been increasing concentration of methane. It has doubled
since pre-industrial times.
y Despite increasing methane emissions, there is a reduction in
anthropogenic emissions.
y The mitigation potential of methane emission varies between
regions and countries with different target sectors in different
countries.
⚪ Fossil fuel industry holds the largest potential for low-cost
methane mitigation potential.

77. Global Methane Other News


Assessment y Global Methane Pledge: Global Methane Pledge was launched at
COP 26 to catalyse action to reduce methane emissions
⚪ Global Methane, Climate and Clean Air (GMCCA) Forum was
sponsored by the Global Methane Initiative (GMI) and the UNEP-
convened Climate and Clean Air Coalition (CCAC).
⚪ Objective: global efforts to reduce methane and other short-
lived climate pollutants.
y Methane and associated concern: Methane (CH4) is a gas that is
found in small quantities in Earth’s atmosphere. It is a powerful
greenhouse gas which is flammable, and is used as a fuel worldwide.
⚪ Source: MethaneLeaks in oil and gas industries, rearing livestock
and the decomposition of waste in landfills.
⚪ Concern: Methane contributes to about one-third of the current
anthropogenic greenhouse gas-driven global warming.
 It is responsible for creating ground-level ozone, a dangerous
air pollutant.
⚪ Currently, only 2% of global climate finance goes to methane.

Climate and Clean Air Coalition


y Established by the governments of Bangladesh, Canada, Mexico,
78. Climate and
Ghana, Sweden and the United States along with UNEP in 2012.
Clean Air
y Objective: To improve air quality and protect the climate through
Coalition
actions to reduce short-lived climate pollutants.
y India is also a member.
y Along with UNEP, it published the Global Methane Assessment.

70 Environment
Objective
y State Energy and Climate Index (SECI) is released by Niti Aayog
to encourage healthy competition among states on different
dimensions of the energy and climate sector.
Parameters
y The State performance is evaluated on the basis of 27 key
performance indicators covered under 6 broad parameters.
⚪ Discom performance
⚪ Access, affordability and reliability
⚪ Clean energy initiatives
⚪ Energy efficiency
79. State Energy & ⚪ Environmental sustainability
Climate Index ⚪ New initiatives
y Based on SECI ranks, the states and UTs are categorised into Front
Runners, Achievers, and Aspirants.
y The overall score for India is 40.6.
y The states have been categorized based on size and geographical
differences as larger states, smaller states, and UTs.
⚪ Gujarat, Kerala, and Punjab have been ranked as the top three
performers in the category of larger states.
⚪ Goa followed by Tripura, and Manipur, emerged as the top-
performing state in the smaller states category.
⚪ Among UTs, Chandigarh, Delhi, and Daman & Diu/Dadra & Nagar
Haveli are the top performers.

y SoIB report is a comprehensive assessment of bird species in India,


driven by citizen science.
y The 2020 report highlights the importance of systematic bird
monitoring and the various threats faced by birds in India.

Findings of SoIB 2023


y As per findings of SoIB 2023, some generalist species (such as
feral Rock Pigeon, Ashy Prinia, Asian Koel, and Indian Peafowl) are
thriving, many others are facing various degrees of decline.
80. State of India’s
y A substantial 60% of species show long-term declines, and 40%
birds 2020 report
are currently in decline.
⚪ The decline is particularly evident among habitat specialists,
especially those in grasslands, open habitats, wetlands, and
woodlands
y Certain dietary groups such as carnivores, insectivores, and
granivores are declining more rapidly than omnivores or fruit- and
nectar-eaters.
y Migratory species and those endemic to the Western Ghats–Sri
Lanka region are also facing significant threats.

Environment 71
y It is released annually by the United Nations Environment
Programme (UNEP).

Objective:
y It records the difference between where greenhouse emissions
are predicted to be in 2030 and where they should be to avert
81. Emissions gap the worst impacts of climate change.
report 2021
Highlights of Emissions Gap Report 2023
y To limit warming to 1.5-2°C, substantial emission cuts of 28-42%
by 2030 are necessary.
y Greenhouse Gas Emissions (GHG) were recorded at 57.4 Gigatonnes
of Carbon Dioxide Equivalent (GtCO2e) in 2022, with a 1.2% increase
from the previous year.

y It is released by World Wide Fund for Nature every two years


y Measures how species are responding to pressure in the environment
due to biodiversity loss and climate change.

82. Living planet Key findings:


report 2020 y 69% decline in the wildlife population of mammals, birds,
amphibians, reptiles, and fish, across the globe in the last 50
years.
y Highest decline was observed in Latin America and the Caribbean
region.

The fourth and final installment of the sixth assessment report (AR6)
by the Intergovernmental Panel on Climate Change was released.

Key Takeaways of the report


y GreenHouse Gas(GHG) emissions will lead to increasing global
warming in the near term, and it’s likely this will reach 1.5°C
between 2030 and 2035.
83. Intergovernmental y The world is currently at around 1.1°C of warming, and current
Panel on Climate climate policies are projected to increase global warming by 3.2°C
Change: Sixth by 2100.
Assessment y The IPCC has “very high confidence” that the risks and adverse
Report impacts from climate change will escalate with increasing global
warming.
y To keep within the 1.5°C limit, emissions need to be reduced by at
least 43% by 2030 compared to 2019 levels and at least 60% by
2035.
y Losses and damages will disproportionately affect the poorest and
most vulnerable populations, particularly those in Africa and least-
developed countries, creating more poverty.

72 Environment
y Tracked climate finance for mitigation falls short of the levels
needed to limit warming to below 2°C or to 1.5°C across all sectors
and regions.
y Public and private finance flows for fossil fuels are still greater
than those for climate adaptation and mitigation.
y Prioritizing equity, social justice, inclusion, and just transition
processes would enable ambitious climate mitigation actions and
climate-resilient development.

IPCC
y The Intergovernmental Panel on Climate Change is a United Nations
body for assessing the science related to climate change.
y It was created in 1988 by the World Meteorological Organization
(WMO) and the United Nations Environment Programme (UNEP).
y Objective – To provide governments, at all levels, with scientific
information that they can use to develop climate policies.
y The IPCC does not conduct its own research. Thousands of people
from all over the world voluntarily contribute to the work of the
IPCC.

In the recent 2023 survey, it is revealed that Duck species sighted in


previous surveys were missing this time.
Major Highlights of the Survey
y Major Missing Species: Duck species like Northern Shoveler,
Common teal and Eurasian wigeon, sighted in the previous surveys,
were totally missing this time around.
y Climate Change Impact: Climate change has affected the number
of birds visiting the region. However, the precise impact of climate
change on bird migration requires more detailed studies.

Highlights of the Survey


y Major Missing Species:
84. IPCC: Asian
⚪ Duck species like Northern Shoveler, Common teal and Eurasian
Waterbird Census
wigeon, sighted in the previous surveys, were totally missing
this time around.
y Climate Change Impact:
⚪ Climate change has affected the number of birds visiting the
region. However, the precise impact of climate change on bird
migration requires more detailed studies.

What is the Asian Waterbird Census?


y It is a citizen-science programme supporting conservation and
management of wetlands and waterbirds worldwide.
⚪ AWC is part of the global International Waterbird Census (IWC)
coordinated by Wetlands International.

Environment 73
y AWC runs in parallel with other regional programmes of the IWC in
Africa, Europe, West Asia, the Neotropics and the Caribbean.
y AWC in India:
⚪ It was initiated in the Indian subcontinent in 1987 and since
then has grown rapidly to cover major regions of Asia, from
Afghanistan eastwards to Japan, Southeast Asia and Australasia.
⚪ In India, AWC is jointly coordinated by the Bombay Natural
History Society (BNHS) and Wetlands International.

According to a new report, Cloud Forest Assets Financing is a Valuable


Nature-Based Solution released by Earth Security – a global nature-
based asset management advisory firm.
Cloud forests:
y Cloud forests are the forests that are on top of the tropical
mountains.
y Cloud forests, also called Montane rainforests, are vegetation
of tropical mountainous regions in which the rainfall is often
heavy and persistent condensation occurs because of cooling of
moisture-laden air currents deflected upward by the mountains.
y These forests occupy a limited area and are under great threat
and their hydrological function is of existential value to millions of
people living downstream.
y Just 25 countries hold 90 per cent of the world’s cloud forests
that capture moisture from the air, providing fresh and clean water
to people and industries below.
y Countries build hydroelectric power plants which use water from
these forests.
85. Cloud Forest
Assets Countries having cloud forests:
y These twenty-five countries are Indonesia, Tanzania, Democratic
Republic of Congo, Colombia, Peru, Venezuela, Mexico, Papua New
Guinea, Brazil, Ethiopia, Ecuador, Cameroon, Bolivia, China, Laos,
Kenya, Malaysia, Angola, Uganda, Madagascar, Philippines, Gabon,
Vietnam, Republic of Congo and Myanmar.

Significance:
y Their hydrological function is of existential value to millions of
people living downstream.
y They capture moisture from the air, providing fresh and clean
water to people and industries below.
y These 25 countries have around 979 hydropower dams and around
half of them use water from the cloud forest.

Cloud forest bond:


y The bond will provide these governments with financial actors like
philanthropy, public finance and private investment to capture the
economic value of the ecosystem services of the cloud forests.

74 Environment
y
y Such a tool will encourage carbon storage and provide funding
to set up sovereign-level carbon finance schemes as well as
payments for ecosystem services.
y The Cloud Forest Bonds will allow the developing countries to
improve their debt position and fund the creation of new, long-
term income streams from services provided by nature.
y These bonds can be in the form of new bond issuances, debt-swaps
and results-based financing instruments, which are matched to
the circumstances of each of the twenty-five countries.
y A Cloud Forest 25 (CF25) Investment Initiative to establish a
collective of all 25 countries that have high cloud forest to accelerate
the international application of market templates and aggregate
the blended finance and data needed to achieve solutions at scale

-World Meteorological Organization (WMO) released the State of


Climate Services report 2021. It focuses on Terrestrial Water Storage.

About the Report


y It focuses on key climate indicators – greenhouse gasses,
temperatures, sea level rise, ocean heat and acidification, sea ice
and glaciers. It also highlights the impacts of climate change and
extreme weather.
y It shows the planetary scale changes on land, in the ocean and
in the atmosphere caused by record levels of heat-trapping
greenhouse gasses.

Findings:
86. State of climate y Increase in Greenhouse Gases (GHGs) emissions: Global GHGs
services report emissions continued to increase in 2022. Carbon dioxide is at 149%
2021 of pre-industrial levels, Methane is at 262% of pre-industrial levels,
Nitrous oxide is at 124% of pre-industrial levels.
⚪ The annual increase of methane was 18 ppb from 2020 to 2021.
This is the largest increase on record.
y High Global Mean Temperature: In 2022, the planet was 1.15 ± 0.13
°C warmer than the pre-industrial (1850-1900) average, making
the last 8 years the warmest on record.
⚪ Despite cooling La Niña conditions , 2022 was the 5th or 6th
warmest year.
y Above Normal Precipitation: In 2022, large areas with above normal
precipitation included large parts of Asia and the south-west
Pacific, areas of northern South America and the Caribbean, the
eastern Sahel region, parts of southern Africa, Sudan, and eastern
Europe.

Environment 75
y Ocean Heat Content: As GHGs accumulate in the atmosphere,
temperatures warm on land and in the ocean. It is expected that
the ocean will continue to warm well into the future.
y Rise in Sea Level: In 2022, global mean sea level continued to rise.
The sea has risen approximately 3.4 ± 0.3 mm per year over the
past 30 years .
y Ocean Acidification: Global mean ocean pH has been steadily
declining at rates not seen for at least the past 26,000 years.
y Sea Ice Extent: Arctic sea-ice extent was below the long-term
average for most of the year.
y Antarctic sea-ice extent dropped to the lowest level and almost
1 million km 2 below the long-term (1991-2020) mean. The total
extent of Antarctic sea ice continued to be below average.

World Meteorological Organization


y Originated from the International Meteorological Organization
(IMO), which was founded in 1873.
y It was established in 1950.
y It is the specialized agency of the United Nations for
meteorology(weather and climate), operational hydrology and
related geophysical sciences.
y Currently it has a membership of 187 countries.
y Governance Structure:
y Its supreme body is the World Meteorological Congress, which
consists of representatives of all members.
y It meets at least every four years to set general policy and adopt
regulations.
y A 36-member Executive Council meets annually and implements
policy.
y Headquarters: Geneva, Switzerland.

y Recently, the United Nations Environment Programme (UNEP) has


released a comprehensive assessment named ‘From Pollution to
Solution: a global assessment of marine litter and plastic pollution’.

Key Findings;
87. Global y Tripled Amount of Plastic:
assessment on ⚪ The amount of plastics in the oceans has been estimated to be
marine litter and around 75-199 million tonnes at present.
plastic pollution ⚪ Without meaningful action, emissions of plastic waste into
aquatic ecosystems are projected to nearly triple by 2040.
y Biodegradable Plastic:
⚪ Plastics labeled as biodegradable may take hundreds of years to
degrade in the oceans; litter poses similar risks to individuals,
biodiversity and ecosystem functioning.

76 Environment
y Main Source of Marine Litter:
⚪ The main sources of marine litter and plastic pollution are
land-based.
⚪ Approximately 7,000 million of the estimated 9,200 million
tonnes of cumulative plastic production between 1950 and
2017 became plastic waste.
y Alteration in Global Carbon Cycle:
⚪ Plastic can also alter global carbon cycling through its effect
on plankton and primary production in marine, freshwater and
terrestrial systems. Greenhouse gas emission from plastics:

Plastic :
y Plastic is a synthetic organic polymer made from petroleum
with properties ideally suited for a wide variety of applications,
including packaging, building and construction, household and
sports equipment, vehicles, electronics and agriculture.
y Plastic is cheap, lightweight, strong and malleable. Over 300 million
tons of plastic are produced every year.
y Sources of Plastic:
⚪ The main sources of marine plastic are land-based, from urban
and storm runoff, sewer overflows, beach visitors, inadequate
waste disposal and management, industrial activities,
construction and illegal dumping.

⚪ Ocean-based plastic originates mainly from the fishing industry,


nautical activities and aquaculture.
⚪ Under the influence of solar UV radiation, wind, currents and
other natural factors, plastic fragments into small particles,
termed microplastics (particles smaller than 5 mm) or
nanoplastics (particles smaller than 100 nm).
y Plastic Waste In India:
⚪ Plastic waste contributes about 5-6 per cent of total solid
waste generated in India.
⚪ India consumes about 13 million tonnes of plastic and recycles
only about 4 million tonnes.

Initiatives of India
y The Plastic Waste Management Rules, 2016 clearly stipulate that
urban local bodies (ULBs) should ban less than 50 micron thick
plastic bags and not allow usage of recycled plastics for packing
food, beverage or any other eatables.
⚪ The Rules also require that local bodies should provide separate
collection, storage and processing of plastic waste in their
areas.
y The government has set an ambitious target of eliminating single-
use plastics by 2022.

Environment 77
y India is a signatory to MARPOL (International Convention on
Prevention of Marine Pollution).
y In addition, Prevention of Marine Pollution is also dealt with
by Merchant Shipping Rules, 2009 framed under the Merchant
Shipping Act, 1958.

Global Efforts
y Legal efforts have been made at the international and national
levels to address marine pollution.
y The most important are:
⚪ The 1972 Convention on the Prevention of Marine Pollution by
Dumping Wastes and Other Matter (or the London Convention)

⚪ The 1996 Protocol to the London Convention (the London


Protocol)
⚪ The 1978 Protocol to the International Convention for the
Prevention of Pollution from Ships (MARPOL).
⚪ The United Nations Environment Program (UNEP) considers
plastic marine debris and its ability to transport harmful
substances as one of the main emerging issues affecting the
environment.
⚪ At the 2015 G7 summit in Bavaria, Germany, the risks of
microplastics were acknowledged in the Leaders’ Declaration.
⚪ GloLitter Partnerships (GLP): It is a project launched by the
International Maritime Organization (IMO) and the Food and
Agriculture Organization of the United Nations (FAO) and initial
funding from the Government of Norway.
⚪ Clean Seas Campaign:
 The United Nations Environment Programme launched the
Clean Seas Campaign in 2017
 Aim: The goal was to galvanize a global movement to turn
the tide on plastic by reducing the use of unnecessary,
avoidable and problematic plastics including single-use
plastics and phasing out intentionally added microplastics.
 Since then, 63 countries have pledged to do their part
to improve plastics management through, among other
measures, reducing the prevalence of single-use plastic
products.
 The campaign will now highlight source to sea issues and
solutions and call for urgent global action.
 The Campaign contributes to the goals of the Global
Partnership on Marine Litter and the New Plastics Economy
Global Commitment.

78 Environment
Miscellaneous

y Mawmluh Cave in Meghalaya, locally known as Krem Mawmluh, has


been listed as UNESCO’s one of the ‘First 100 IUGS (International
Union of Geological Sciences) Geological Sites’ in the world.
⚪ The caves are located in East Khasi Hill district.

Key features
88. Mawmluh Cave y It is believed to be the fourth longest cave in the Indian subcontinent
with a total length of 7 km of cave passages.
y The cave contains passages, calcite formations of various kinds,
and massive caverns.
y The cave remains in darkness with only ¼th of the cave receiving
sunlight.
y The pool inside the cave is formed from five different rivers that
pass through the cave.

y The year 2023 has been recognised as the International Year of


Millets or IYM2023 for awareness about health and nutritional
benefits of millets.

About Millets
y Millets are small-grained, annual, warm-weather cereals belonging
to the grass family.
y Different millets are:
⚪ Jowar (Sorghum), Bajra (Pearl Millet) and Ragi (Finger millet) are
89. International Year
the important millets cultivated in India.
of Millets (IM)
⚪ Small Millets such as Proso (Cheena), Kodo (Kodra, Arikelu), Fox
2023
tail (Kangni/Korra), Barnyard (Varai, Sawa), Little millet (Kutki)
are also grown in our country.
y Millets are the staple crops of the semi arid tropics, as other food
crops cannot be cultivated in that terrain due to low rainfall and
poor soil fertility.
y They also have higher nutrient content compared to major cereal
crops and ensure food and nutrition security. Further, millets are
tolerant to drought and other extreme weather conditions and
hence are endemic to such geographies.

Farmers rejected the direct-seeded rice (DSR) technique for


transplanting paddy in Punjab.

About direct-seeded rice (DSR)


90. Direct-seeded
y DSR refers to the process of establishing a rice crop from
Rice (DSR)
seeds sown in the field rather than by transplanting seedlings from
the nursery.
⚪ It is a water-saving method of sowing paddy.
⚪ DSR is also known as broadcasting seed technique.

Environment 79
y Other methods of rice cultivation:
⚪ Seed transplantation
⚪ System of rice Intensification: Done in Kerala

y EESL recently pledged support to the implementation of Energy


Efficiency (EE) projects in Andhra Pradesh aimed at mitigating the
impact of climate change, especially in the MSME sector.

About Energy efficiency services limited (EESL)


y EESL is promoted by the Ministry of Power, Government of India, as
a Joint Venture of four reputed public-sector undertakings:
⚪ NTPC Limited,
⚪ Power Finance Corporation Limited,
91. Energy efficiency ⚪ REC Limited and POWERGRID Corporation of India Limited.
services limited y It was registered under the Companies Act, 1956.
(EESL) y EESL is also leading the market-related activities of the National
Mission for Enhanced Energy Efficiency (NMEEE).

Objective:
y EESL was formed to create and sustain market access to
energy efficient technologies, particularly in public facilities like
municipalities, buildings, agriculture, industry etc.
y To implement several schemes of the Bureau of Energy Efficiency,
Ministry of Power and Ministry of New & Renewable Energy,
Government of India.

y India has advanced the target of 20% ethanol blending in petrol


( E20), by five years to 2025, from 2030 by making amendments to
National Policy on Biofuels, 2018

Ethanol Blending
y It is defined as a blended motor fuel containing ethyl alcohol that
92. Ethanol blending
is at least 99% pure, derived from agricultural products, and
blended exclusively with gasoline.
⚪ It is considered a renewable fuel as it is plant-based.
y Ethanol production is allowed from the following: sugarcane-based
raw materials, sugarcane juice / sugar / sugar syrup, surplus rice
with Food Corporation of India (FCI) and Maize

y Limit of growth theory says nature has set a limit beyond the limit
development cannot possible.
y Developed by: D. H. Meadows on the basis of system dynamic
principle.
93. Limits to growth
Basic assumptions:
y We have limited resources available on earth.
y Limited agricultural land

80 Environment
y Earth has a limited capacity to consume the pollution
y Importance of technology & innovation to change productivity.
y No importance to recycling & pollution control mechanism

y The Emissions Database for Global Atmospheric Research is A


global emission inventory of GHGs and air pollutants from the food
systems.
y EDGAR-FOOD represents the first database consistently covering
each stage of the food chain for all countries with yearly frequency
94. Edgar food for the period 1990-2015.
y Objective: aid the understanding of the activities underlying the
energy demand and use, agriculture and land use change emissions
associated with the production, distribution, consumption and
disposal of food through the various stages and sectors of the
composite global food system.

y It is a novel method of assessing biodiversity overall and species


presence

Working
95. Environmental
y Samples are taken from the environment via water, sediment or
dna
air from which DNA is extracted, and then amplified using general
metabarcoding
or universal primers in polymerase chain reaction and sequenced
(edna)
using next-generation sequencing to
⚪ eDNA describes the genetic material present in environmental
samples such as sediment, water, and air, including whole cells,
extracellular DNA and potentially whole organisms

y It is a statutory body constituted under Wildlife Protection Act, 1972


⚪ It was established in 2005 following the recommendations of
the Tiger Task Force.

Objectives:
y Strengthening tiger conservation, as per powers and functions
96. National tiger assigned to it
conservation
authority (NTCA) Functions:
y Providing statutory authority to Project Tiger
y Addressing livelihood interests of local people in areas surrounding
the Tiger Reserves
y Approve the Tiger Conservation Plan prepared by the State
Governments
y Overarching supervisory/coordination role

Environment 81
y An organism that causes ecological or economic harm in a new
environment where it is not native.
y Examples in India include:
⚪ Parthenium (came from wheat imported from U.S. in 1950s)
97. Invasive Species ⚪ lantana (brought by the British as ornamental plants from
South America) threatens more than 40% of India’s tiger
reserves.
y Other examples include Vilayati Kikar, American bullfrog, Senna
spectabillis, Red-eared slider turtle

y Recently, Parliament passed the Indian Antarctic Bill, 2022.


⚪ It is an important step forward in India’s engagement with the
gigantic continent which began way back in February 1956.
y About the Bill:
⚪ It is the first domestic legislation with regard to Antarctica in
India.
⚪ Applicable to Indian citizens as well as foreign citizens.

y Objective:
⚪ To demilitarise Antarctica; use of Antarctica for peaceful
purposes; promote international scientific cooperation in
Antarctica.
⚪ The Bill seeks to give effect to the Antarctic Treaty, the
Convention on the Conservation of Antarctic Marine Living
Resources, and the Protocol on Environmental Protection to
the Antarctic Treaty.

98. The Indian Key Features


Antarctic Act y Prohibit Indian expedition to Antarctica without permit or written
2022 authorisation of another party to Antarctic Treaty.
y Extend jurisdiction of Indian courts to Antarctica and lays out
penal provision for crimes on the continent by Indian citizens,
foreign citizens who are part of Indian expeditions.
y The act directs creating a fund called the Antarctic fund that will
be used for protecting the Antarctic environment.
y The Bill also establishes a ‘Committee on Antarctic Governance
and Environmental Protection.’
y Prohibits mining, dredging and activities that threaten the pristine
conditions of the continent.

What is the Antarctica Treaty


y The Treaty covers the area south of 60°S latitude.
y Objective: To demilitarize Antarctica; use for peaceful purposes &
resolve disputes.
y Signed in 1959 by 12 countries: Argentina, Australia, Belgium, Chile,
France, Japan, New Zealand, Norway, South Africa, USSR, the UK
and the USA, and came into force in 1961.

82 Environment
⚪ India signed the Antarctic Treaty in 1983.
y Convention on the Conservation of Antarctic Marine Living
Resources (CCAMLR): Set up in 1980 for the protection and
preservation of the Antarctic.
y Madrid Protocol: Designates Antarctica as a “natural reserve,
devoted to peace and science” (Signed by India).

Key Facts about Antarctica:


y The world’s southernmost and fifth-largest continent has the
geographic South Pole.
y The continent is divided into East Antarctica (which is largely
composed of a high ice-covered plateau) and West Antarctica
(which is largely an ice sheet covering an archipelago of mountainous
islands).
y Antarctica is a unique continent in that it does not have a native
population. There are no countries in Antarctica, although seven
nations claim different parts of it: New Zealand, Australia, France,
Norway, the United Kingdom, Chile, and Argentina.
y The Indian Antarctic program (1981) has completed 41 scientific
expeditions and built three permanent research base stations in
Antarctica.
⚪ DakshinGangotri (1983), Maitri (1988) and Bharati (2012).
⚪ ‘Himadri’ station in Svalbard, above the Arctic circle.
y The National Centre for Polar and Ocean Research (NCPOR), Goa—
an autonomous institute under the Ministry of Earth Sciences—
manages the entire Indian Antarctic program.
y Longest River: Onyx.
y Largest Lake: Vostok, is one of the largest subglacial lakes in the world.

Recently, powerful tremors were felt in Turkey after an earthquake of


magnitude 7.8 struck along a well-known fault line called the Anatolia
tectonic block.

What Makes Turkey Prone to Earthquakes?


y In the Eastern Mediterranean region comprising Turkey, Syria
and Jordan, tectonics are dominated by complex interactions
between the African, Arabian, and Eurasian tectonic plates, and
99. Anatolian the Anatolian tectonic block.
Plateau Issue y Turkey sits on the Anatolian tectonic plate, which borders two
major faults, the North Anatolian Fault (NAF) that cuts across the
country from west to east, and the East Anatolian Fault (EAF) in
the southeast.
⚪ The NAF line is the meeting point of the Eurasian and Anatolian
tectonic plates that is known to be “Particularly Devastating”.
 NAF is right-lateral strike-slip structure in northern Turkey
accommodating much of the translational motion of the
Anatolia block westwards with respect to Eurasia and Africa.

Environment 83
 The EAF is the tectonic boundary between the Anatolian
Plate and the northward-moving Arabian Plate. It runs 650
kilometers from eastern Turkey and into the Mediterranean.
y In addition to this, the Aegean Sea Plate, located in the eastern
Mediterranean Sea under southern Greece and western Turkey, is
also a source of seismic activity in the region.
y According to one estimate, almost 95% of Turkey’s land mass is
prone to earthquakes, while about a third of the country is at high
risk, including the areas around the major cities of Istanbul and
Izmir and the region of East Anatolia.

Wildlife Justice Commission (WJC), a nonprofit organization dedicated


to combating organized crime, has released a new report titled
Convergence of Wildlife Crime with Other Forms of Organised Crime:
A 2023 Review.
y It is a follow-up to the first report published in 2021, which
mentioned 12 case studies linking wildlife trafficking with human
trafficking, fraud, migrant smuggling , illicit drugs corruption and
money laundering.
y The report also reveals the environmental crime of illegal sand
mining for the first time.

Wildlife Justice Commission Report:


y The report reveals close connections between wildlife trade and
various criminal activities, including protection rackets, extortion,
murder, money laundering, illicit drugs, tax evasion, and corruption.
y This report, titled “Convergence of wildlife crime with other forms
100. Illegal Wildlife of organized crime: A 2023 Review,” is a follow-up to the 2021
Trafficking report, which highlighted 12 case studies demonstrating links
between wildlife trafficking and human trafficking, fraud, migrant
smuggling, illicit drugs, corruption, and money laundering.
y However, it is the first time that it noted sand-mining as a form of
environmental crime.

Wildlife Trade:
y It Involves the sale and exchange of wild animals and plants,
including live specimens, parts, derivatives, or transformed
products.
y Occurs at various levels, from local to global.
y Regulated internationally by CITES (Convention on International
Trade in Endangered Species of Wild Fauna and Flora).
y Wildlife Trade in India: International wildlife trafficking into and
out of India mainly occurs through either the Northeast or through
airports. Chennai and Mumbai airports are major hubs for this
illegal activity.

84 Environment
Major Trafficking Routes (The 2018 TRAFFIC report In Plane Sight)
y Northeastern Borders: Rhino horns, tiger parts, and pangolin scales
are trafficked extensively across the borders with Nepal, Myanmar,
and China, with transit sites in cities like Dimapur, Guwahati, and
Imphal.
y India-Bangladesh Border: The trafficking of birds and reptiles is
prevalent along the India-Bangladesh border.

Global Cooperation in Combating Wildlife Crime


y International Consortium on Combating Wildlife Crime (ICCWC)
⚪ Collaborative effort of five inter-governmental organizations:
CITES Secretariat, INTERPOL, the United Nations Office on
Drugs and Crime (UNODC), World Bank, and World Customs
Organization (WCO).
⚪ Provides coordinated support to national wildlife law
enforcement agencies and regional networks.

The blooms of Noctiluca Scintillans, commonly known as “sea sparkle”


are being witnessed along the coasts of Maharashtra and Karnataka.

Noctiluca Scintillans
y Scintillans is a bioluminescent species that brightens seawater
during the night.
y It grazes on other micro-organisms such as larvae, fish eggs, and
diatoms. But the unicellular phytoplankton that lives inside it can
photosynthesize, turning sunlight into energy.
y They helped their host cell survive even when food was scarce.
y Thus, N. Scintillans acts as both a plant and an animal

Threats posed
101. Sea Sparkle
y According to marine experts, the phenomenon is an indicator of
climate change.
y While smaller blooms may be harmless, slow-moving larger blooms
may have an impact on deep-sea fishes.
y The toxic blooms of N. Scintillans were linked to massive fish and
marine invertebrate kills.
y Though the species does not produce a toxin, it was found to
accumulate toxic levels of ammonia, which is then excreted into
the surrounding waters, possibly acting as the killing agent in
blooms.
y They have displaced microscopic algae called diatoms, which form
the basis of the marine food chain. This has deprived food for the
planktivorous fish.

Environment 85
Previous Years Questions

1. With reference to ‘Eco-Sensitive Zones’, (d) Sultanpur National Park


which of the following statements is/are Answer: (b)
correct? (2014) Option (b) is correct: Keibul Lamjao National
1. Eco-Sensitive Zones are the areas Park is the only floating park in the world.
that are declared under the Wildlife The Park is located in the southwestern
(Protection) Act, 1972. part of the Loktak Lake. The Loktak Lake
2. The purpose of the declaration of Eco has a unique ecosystem called ‘Phumdi’ (a
Sensitive Zones is to prohibit all kinds of Manipuri word meaning floating mats of soil
human activities in those zones except and vegetation).
agriculture. y The largest area of the Phumdi in the
Select the correct answer using the code Loktak lake is in the Keibul Lamjao
given below: National Park, which is home to Manipur
(a) 1 only brow-antlered deer, also popularly
(b) 2 only known as the Sangai. The habitat
(c) Both 1 and 2 exclusively consists of floating meadows
(d) Neither 1 nor 2 and an elevated strip of hard ground
Answer: (d) that dissects the park into northern
Statement 1 is not correct: Eco-Sensitive and southern zones. Loktak Lake is the
Zones or Ecologically Fragile Areas are largest natural freshwater lake in the
areas within 10 km around Protected Areas, north-eastern region of India.
National Parks and Wildlife Sanctuaries. Additional Information:
ESZs are notified by the Ministry of y Bhitarkanika is India’s second-
Environment Forest and Climate Change largest mangrove ecosystem after
(MoEFCC), Government of India under the Sunderbans. The National Park
Environment Protection Act 1986. is essentially a network of creeks
Statement 2 is not correct: Commercial and canals which are inundated with
mining, setting of sawmills and industries waters from rivers Brahmani, Baitarani,
causing pollution, commercial use of Dhamra and Patasala forming a unique
firewood and major hydro-power projects, ecosystem.
are prohibited in such areas. Activities y Keoladeo National Park is in the State
permitted in the areas include ongoing of Rajasthan. Formerly known as the
agriculture and horticulture practices by Bharatpur Bird Sanctuary, the Keoladeo
local communities, rainwater harvesting, National Park is recognised as one of the
organic farming, adoption of green world’s most important bird breeding
technology and use of renewable energy and feeding grounds. In 1982, Keoladeo
sources. was declared a national park and then
later listed as a World Heritage Site by
2. Which of the following National Parks is UNESCO in 1985. It was also known as
unique in being a swamp with floating the breeding ground for the rare and
vegetation that supports a rich biodiversity? elusive Siberian crane.
(2015) y Sultanpur National Park & Bird
(a) Bhitarkanika National Park Sanctuary is in Gurgaon district of
(b) Keibul Lamjao National Park Haryana. The Sultanpur National Park is
(c) Keoladeo Ghana National Park lush with trees, shrubs, and clusters of
bougainvillaea. Ramsar Convention.
y India signed the Ramsar Convention on
3. If a wetland of international importance is 1 February 1982.
brought under the ‘Montreux Record’, what y There are 42 Ramsar sites in India. Chilika
does it imply? (2014) Lake (Odisha) and Keoladeo National
(a) Changes in ecological character have Park (Rajasthan) were recognized as the
occurred, are occurring or are likely first Ramsar Sites of India.
to occur in the wetland as a result of y Montreux Record sites in India: Keoladeo
human interference. National Park and Loktak Lake.
(b) The country in which the wetland is
located should enact a law to prohibit 4. With reference to the usefulness of the
any human activity within five kilometres by- products of the sugar industry, which
from the edge of the wetland. of the following statements is/are correct?
(c) The survival of the wetland depends on (2013)
the cultural practices and traditions of 1. Bagasse can be used as biomass fuel
certain communities living in its vicinity for the generation of energy.
and therefore the cultural diversity 2. Molasses can be used as one of the
therein should not be destroyed. feedstocks for the production of
(d) It is given the status of ‘World Heritage synthetic chemical fertilisers.
Site’. 3. Molasses can be used for the production
Answer: (a) of ethanol.
The Montreux Record was established by Select the correct answer using the code
the recommendation of the Conference given below:
of the Contracting Parties (1990). During (a) 1 only
the fourth meeting held at Montreux, (b) 2 and 3 only
Switzerland in 1990, it was decided that (c) 1 and 3 only
mere listing of wetlands as Ramsar sites (d) 1, 2 and 3
was not enough. The purpose would be Answer: (c)
fulfilled if the conservation activities could Sugarcane today is considered as one of
be prioritised where ecological damage has the best converters of solar energy into
begun or is likely to occur. biomass and sugar. It is a rich source of
Option (a) is correct: The Montreux Record food (sucrose, jaggery and syrup), fibre
is a register of wetland sites on the List (cellulose), fodder (green leaves and tops of
of Wetlands of International Importance cane plant, bagasse, and molasses and to
where changes in ecological character have some extent press mud), fuel and chemical.
occurred, are occurring, or are likely to occur The main by-products are bagasse,
as a result of technological developments, molasses and press mud.
pollution or other human interference. It is Statement 1 is correct: Bagasse is a fibrous
maintained as part of the Ramsar List. residue left over after the sugarcane is
Additional Information: crushed. The fibre content of bagasse
y The Convention on Wetlands is the (cellulose) is used in cellulose industries
intergovernmental treaty that provides like the pulp, paper, particle boards, cattle
the framework for the conservation feed etc. Bagasse can be used as biomass
and wise use of wetlands and their fuel for the generation of energy.
resources. The Convention was adopted Statement 2 is not correct: The main
in the Iranian city of Ramsar in 1971 and products that can be produced from
came into force in 1975. molasses on commercial scale are ethyl
y There are 171 contracting parties to the alcohol, citric acid, lactic acid, cattle feed,

Previous Years Questions 87


oxalic acid, baker’s yeast, monosodium on animal welfare laws and policy making.
glutamate, torula yeast, lysine, acetone- Statement 2 is correct: National Tiger
butanol-alcohol. In addition to this, many Conservation Authority (NTCA) is a
alcohol-based chemicals like acetic acid, Statutory Body under the Ministry of
acetic anhydride, acetone, ethyl acetate, Environment, Forests and Climate Change.
ethyl benzene, styrene, poly-styrene, poly- It was established in 2005 following the
ethylene and synthetic rubber are also recommendations of the Tiger Task Force. It
produced using cane molasses. It is also was constituted under enabling provisions
used for the production of organic fertiliser. of the Wildlife (Protection) Act, 1972, as
Statement 3 is correct: Sugarcane ethanol amended in 2006, for strengthening tiger
is an alcohol-based fuel produced by conservation, as per powers and functions
the fermentation of sugarcane juice and assigned to it.
molasses. Because it is a clean, affordable Statement 3 is correct: National Ganga
and low-carbon biofuel, sugarcane ethanol River basin authority was constituted
has emerged as a leading renewable fuel under the Environment Protection Act in
for the transportation sector. In India, 2009 as a Statutory Body headed by the
Bioethanol can be produced from multiple Prime Minister. The main functions are
sources like sugar containing materials, conservation and abatement of pollution of
starch containing materials, cellulose Ganga, the national river. Chief Ministers of
and lignocelluloses material including a few states through which the river passes
petrochemical route. However, the are also members.
present policy of Ethanol Blended Petrol
(EBP) Programme allows bioethanol to be 6. Why is a plant called Prosopis juliflora
procured from non-food feedstock like often mentioned in the news? (2018)
molasses, cellulose and lignocelluloses (a) Its extract is widely used in cosmetics.
material including petrochemical route. (b) It tends to reduce the biodiversity in the
Similarly, biodiesel can be produced from area in which it grows.
any edible/non-edible oil. (c) Its extract is used in the synthesis of
pesticides.
5. Consider the following statements: (2014) (d) None of the above
1. Animal Welfare Board of India was Answer: (b)
established under the Environment Option (b) is correct: Prosopis juliflora
(Protection) Act, 1986. (vilayati kikar or vilayati babul), an exotic
2. National Tiger Conservation Authority is tree, is one of the top invaders in India.
a Statutory Body. It was introduced into India from South
3. The National Ganga River Basin Authority America and which has become naturalised
is chaired by the Prime Minister. all-over north India. This tree is also
Which of the statements given above is/are affecting the nesting success of birds. It
correct? tends to reduce the biodiversity in the area
(a) 1 only in which it grows.
(b) 2 and 3 only Invasive plants are those plants which-
(c) 2 only spread to regions other than their native and
(d) 1, 2 and 3 thrive to the extent of becoming a threat to
Answer: (b) local species. Alien invasive species occur
Statement 1 is not correct: Animal Welfare in all groups of plants and animals. They
Board of India is a Statutory advisory Body include competitors, predators, pathogens,
created under the Prevention of Cruelty to and parasites. They have invaded almost
Animals Act. It was set up in 1962 to advise every type of native ecosystem and have

88 Previous Years Questions


caused hundreds of extinctions. the pre-industrial level, what can be its
possible impact/impacts on the world?
7. The Himalayan Range is very rich in species (2014)
diversity. Which one among the following 1. Terrestrial biosphere tends toward a net
is the most appropriate reason for this carbon source.
phenomenon? (2011) 2. Widespread coral mortality will occur.
(a) It has a high rainfall that supports 3. All the global wetlands will permanently
luxuriant vegetative growth. disappear.
(b) It is a confluence of different 4. Cultivation of cereals will not be possible
biogeographical zones. anywhere in the world.
(c) Exotic and invasive species have not Select the correct answer using the code
been invasive species and have not given below:
been introduced in this region. (a) 1 only
(d) It has less human interference. (b) 1 and 2 only
Answer: (b) (c) 2, 3 and 4 only
The Himalayas have three biogeographical (d) 1, 2, 3 and 4
zones which have different biodiversity Answer: (b)
as: Northwest Himalayas, West Himalayas, Impact of the global temperature increases
Central Himalayas, and East Himalayas, beyond 30C above the pre-industrial level:
which together constitute about 6.4% of y Melting of ice and emission of carbon
the country’s area. It is a confluence of tapped beneath them will make a
different biogeographical zones due to large carbon source instead of sink.
which the Himalayan region is very rich in (Statement 1 is correct)
species diversity. y Global warming has affected the coral
reefs that can lead to a loss of plant
8. “Biorock technology” is talked about in and animal lives. Increase in global
which one of the following situations ? temperatures has made the fragility of
(2022) coral reefs even worse. Both aquatic and
(a) Restoration of damaged coral reefs terrestrial ecosystems will be affected.
(b) Development of building materials using (Statement 2 is correct)
plant residues y Rising temperatures and acidity within
(c) Identification of areas for exploration/ our oceans are contributing to extreme
extraction of shale gas coral bleaching events, like the 2016
(d) Providing salt licks for wild animals in event that destroyed more than one-
forests/protected areas third of the Great Barrier Reef.
Answer: (a) y Wetlands are drying up and shrinking
Option (a) is correct: Biorock technology around the world. (Statement 3 is not
greatly accelerates coral settlement, correct)
growth, healing, survival, and resistance y Cultivation of crops will face shifts in
to environmental stresses such as high terms of location e.g., wheat cultivation
temperature, sediment, and pollution. All will shift northwards. Also, productivity
other marine organisms examined also will be affected. (Statement 4 is not
benefit. correct)
y There are droughts at some places and
9. The scientific view is that the increase in floods at some. This climatic imbalance
global temperature should not exceed 20C is the result of global warming.
above pre-industrial level. If the global y Global warming leads to a change in
temperature increases beyond 30C above the patterns of heat and humidity. This

Previous Years Questions 89


has led to the movement of mosquitoes 5. Mercury
that carry and spread diseases. 6. Lead
y Due to an increase in floods, tsunamis 7. Plutonium
and other natural calamities, the average Select the correct answer using the code
death toll usually increases. Also, such given below:
events can bring about the spread of (a) 1, 3, 4, 6, and 7 only
diseases that can hamper human life. (b) 1, 2, 3, 5, and 6 only
(c) 2, 4, 5, and 7 only
10. Due to improper/indiscriminate disposal (d) 1, 2, 3, 4, 5, 6 and 7
of old and used computers or their parts, Answer: (b)
which of the following are released into Electronic waste (e-waste) typically
the environment as e-waste? (2013) includes discarded computer monitors,
1. Beryllium motherboards, mobile phones and chargers,
2. Cadmium compact discs, headphones, television sets,
3. Chromium air conditioners, and refrigerators.
4. Heptachlor Option (b) is correct:

11. In India, ‘extended producer responsibility’ was introduced as an important feature in which
of the following? (2019)
(a) The Bio -medical Waste (Management and Handling) Rules, 1998
(b) The Recycled Plastic (Manufacturing and Usage) Rules, 1999
(c) The e -Waste (Management and Handling) Rules, 2011
(d) The Food Safety and Standard Regulations, 2011

90 Previous Years Questions


Answer: (c) up to 70%, (elimination of carbon monoxide
Extended producer responsibility is a practice emissions do not happen by using H-CNG)
and a policy approach in which producers besides enabling up to 5% savings in fuel.
take responsibility for management of the Statement 2 is correct: H-CNG as fuel also
disposal of products they produce once reduces carbon dioxide and hydrocarbon
those products are designated as no longer emissions.
useful by consumers. Statement 3 is correct: Worldwide Hydrogen
Collection mechanism-based approach has is being blended (20-30%) with natural gas
been adopted, which includes collection and then compressed to dispense into
centres, collection points, take back vehicles. The US, Brazil, Canada, and South
systems etc for collection of e-waste Korea have all conducted trials and found
by producers under Extended Producer that they get reduction in emissions from
Responsibility. buses using H-CNG.
Option (c) is correct: In India, ‘extended Statement 4 is not correct: Physical
producer responsibility’ was introduced blending of CNG and hydrogen involves a
as an important feature in the e -Waste series of energy-intensive steps that would
(Management and Handling) Rules, 2011. make H-CNG more expensive than CNG.
Additional Information:
12. In the context of proposals to the use of Compact Reforming Process’ directly
hydrogen - enriched CNG (H-CNG) as fuel produces a Hydrogen-CNG mixture from
for buses in public transport, consider the natural gas, using a single step. The cost
following statements: (2019) of production is significantly lower than
1. The main advantage of the use of H-CNG physical blending.
is the elimination of carbon monoxide
emissions. 13. With reference to ‘Global Environment
2. H-CNG as fuel reduces carbon dioxide Facility’, which of the following statements
and hydrocarbon emissions. is/are correct? (2014)
3. Hydrogen up to one -fifth by volume can (a) It serves as a financial mechanism for
be blended with CNG as fuel for buses. ‘Convention on Biological Diversity’ and
4. H-CNG makes the fuel less expensive ‘United Nations Framework Convention
than CNG. on Climate Change’.
Which of the statements given above is/are (b) It undertakes scientific research on
correct? environmental issues at global level.
(a) 1 only (c) It is an agency under OECD to facilitate
(b) 2 and 3 only the transfer of technology and funds to
(c) 4 only underdeveloped countries with specific
(d) 1, 2, 3 and 4 aims to protect their environment.
Answer: (b) (d) Both (a) and (b)
CNG is compressed natural gas. With natural Answer: (a)
gas mainly composed of methane, CNG Option (a) is correct: The Global
emits less air pollutants such as carbon Environment Facility (GEF) was established
dioxide, carbon monoxide, nitrogen oxides on the eve of the 1992 Rio Earth Summit.
and particulate matter than petrol or diesel. It is a financial mechanism that provides
Statement 1 is not correct: H-CNG is a blend grants to developing countries for projects
of hydrogen and CNG, the ideal hydrogen that benefit the global environment.
concentration being 18%. Compared to Since then, the GEF has provided more
conventional CNG, use of H-CNG can than $21.1 billion in grants and mobilised
reduce the emission of carbon monoxide an additional $114 billion in co-financing for

Previous Years Questions 91


more than 5,000 projects in 170 countries. (halons), used in fire extinguishers.
It serves as a financial mechanism for y As a result of depletion of the ozone
‘Convention on Biological Diversity’ and layer, more ultraviolet (UV) radiation
‘United Nations Framework Convention on comes to Earth and causes damage
Climate Change’. Through its Small Grants to living organisms. UV radiation
Programme, the GEF has provided support seems responsible for skin cancer in
to more than 25,000 civil society and humans; it also lowers production of
community initiatives in 133 countries. phytoplankton and thus affects other
aquatic organisms. It can also influence
14. Which one of the following is associated the growth of terrestrial plants. A
with the issue of control and phasing out reduction of approximately 5 per cent in
of the use of ozone-depleting substances? the ozone layer was detected from 1979
(2015) to 1990.
(a) Bretton Woods Conference Additional Information:
(b) Montreal Protocol y The Bretton Woods Conference held in
(c) Kyoto Protocol 1944 set up the International Monetary
(d) Nagoya Protocol Fund (IMF) and the World Bank and re-
Answer: (b) established a system of fixed exchange
Option (b) is correct: Recognising the rates.
deleterious effects of ozone depletion, an y The Kyoto Protocol was adopted in Kyoto,
international treaty, known as the Montreal Japan, on 11 December 1997. Due to a
Protocol, was signed in Montreal (Canada) complex ratification process, it entered
in 1987 (effective in 1989) to control the into force on 16 February 2005. The
emission of ozone-depleting substances. Kyoto Protocol commits industrialised
y Ozone depletion refers to the countries to stabilise greenhouse gas
phenomenon of reductions in the emissions based on the principles of
amount of ozone in the stratosphere. The the Convention.
problem of ozone depletion is caused y The Nagoya Protocol on Access to
by high levels of chlorine and bromine Genetic Resources and the Fair and
compounds in the stratosphere. Equitable Sharing of Benefits Arising
y The origins of these compounds are from their Utilisation (ABS) to the
chlorofluorocarbons (CFC), used as Convention on Biological Diversity is
cooling substances in air conditioners a supplementary agreement to the
and refrigerators, or as aerosol Convention on Biological Diversity.
propellants, and bromofluorocarbons

92 Previous Years Questions


Previous Years Questions 93

You might also like